0% found this document useful (0 votes)
153 views130 pages

Paid Orthobullet MCQs - Hand

The document describes a patient presenting with swelling of the distal forearm and little finger along with suspected purulent flexor tenosynovitis in the thumb. The most likely anatomic explanation is a potential space between the fascia of the pronator quadratus and flexor digitorum profundus conjoined tendon sheaths, known as Parona's space, which allows infection to track through and present as a horseshoe abscess.
Copyright
© © All Rights Reserved
We take content rights seriously. If you suspect this is your content, claim it here.
Available Formats
Download as PDF, TXT or read online on Scribd
0% found this document useful (0 votes)
153 views130 pages

Paid Orthobullet MCQs - Hand

The document describes a patient presenting with swelling of the distal forearm and little finger along with suspected purulent flexor tenosynovitis in the thumb. The most likely anatomic explanation is a potential space between the fascia of the pronator quadratus and flexor digitorum profundus conjoined tendon sheaths, known as Parona's space, which allows infection to track through and present as a horseshoe abscess.
Copyright
© © All Rights Reserved
We take content rights seriously. If you suspect this is your content, claim it here.
Available Formats
Download as PDF, TXT or read online on Scribd
You are on page 1/ 130

(SAE11OS.

190) When evaluating a patient with suspected purulent flexor


tenosynovitis in the thumb, the distal forearm and little finger are found to be swollen
as well. The most likely anatomic explanation is the existence of a potential space in
which of the following? Review Topic

1 Through the carpal tunnel

2 Across the midpalmar space

3 Communicating with the subcutaneous tissue

4 Superficial to the distal antebrachial fascia

5 Between the fascia of the pronator quadratus and flexor digitorum profundus conjoined
tendon sheaths

PREFERRED RESPONSE 5

Pyogenic flexor tenosynovitis is an infection within the flexor tendon sheath that can
involve the fingers or thumb. The tendon sheaths begin at the metacarpal neck level
and extend to the distal interphalangeal joint. In the little finger and the thumb, the
sheaths usually communicate with the ulnar and radial bursae, respectively. The
potential space of communication, Parona's space, lies between the fascia of the
pronator quadratus muscle and flexor digitorum profundus conjoined tendon sheaths.
Infection tracking through this space presents as a horseshoe abscess.

(SAE11UE.112) Figures 112a and 112b show the radiographs of a 28-year-old


motorcyclist who sustained a closed hand injury in a collision. What is the most
appropriate definitive treatment? Review Topic

1 Closed reduction and a hand/forearm cast in the intrinsic plus position


2 Closed reduction and a hand splint

3 Primary fusion of the carpometacarpal joints

4 Closed versus open reduction and internal fixation

5 Closed reduction and external fixation


PREFERRED RESPONSE 4

Closed versus open reduction and internal fixation is the most appropriate treatment.
The radiographs show fracture-dislocations of all five carpometacarpal joints. These
injuries are extremely unstable and not amenable to closed (splint or cast) treatment
only. External fixation may be warranted in an open, contaminated injury. Fusion
would be an option if this were a chronic, painful condition on presentation.

(OBQ15.256) A 22-year-old athlete presents with the injury shown in Figure A. What
would be the recommended treatment? Review Topic

1 Observation only
2 Removable wrist splint

3 Long arm cast

4 Open reduction internal fixation through a volar approach

5 Open reduction internal fixation through a dorsal approach

PREFERRED RESPONSE 5

This patient presents with a proximal scaphoid fracture. The most appropriate
treatment would be open reduction internal fixation (ORIF) through a dorsal
approach.

ORIF is indicated in almost all cases of scaphoid proximal pole fractures as these is a
high incidence of avascular necrosis (AVN) and non-union with these fracture
patterns. Literature suggests AVN rates to be of 100% with proximal 5th fractures and
and 33% with proximal 3rd frature.
Raskin et al. disccused the dorsal open repair of proximal pole scaphoid fractures.
They state that the ideal approach to manage these fractures is a 3-4 cm dorsal
incision centered over Lister’s tubercle. Care should be taken to preserve the vessels
around the dorsal ridge of the scaphoid, by avoiding capsule stripping in this area.

Rettig et al. report their their outcomes with ORIF of displaced scaphoid waist
fractures. Using internal fixation with K-wires or Herbert screws, 13 of the 14 (93%)
fractures united with an average time to union of 11.5 weeks. They state that ORIF of
acute displaced scaphoid waist fractures restores scaphoid alignment and leads to
predictable union.

Rettig et al. looked at retrograde screw fixation of acute unstable proximal pole
scaphoid fractures for 17 patients. The operative technique consisted of a dorsal
approach to the scaphoid, radius bone grafting, and freehand retrograde Herbert
compression screw fixation. All fractures healed within 13 weeks (average, 10
weeks). No patients developed osteonecrosis or radioscaphoid arthritis.

Figure A shows a minimally displaced proximal 1/3 scaphoid fracture.

Incorrect Answers:
Answers 1-3: Non-operative treatment of proximal scaphoid fractures have shown to
have a high incidence of AVN and non-union. Operative fixation is preferred,
especially in young athletes.
Answer 4: Displaced proximal pole fractures require a dorsal approach because
accurate placement of the screw has been shown to be more reliable with this
approach.

(SAE08AN.39) The arrows in the axial T 1-weighted MRI scan shown in Figure 25
show which of the following structures? Review Topic

1 Ulnar artery and accompanying vein


2 Deep and superficial branches of the ulnar nerve

3 Radial and ulnar digital nerves to the little finger

4 Palmar cutaneous and thenar motor branch of the median nerve

5 Dorsal cutaneous branch of the ulnar nerve and common digital artery to the fourth web
PREFERRED RESPONSE 2

The arrows in the figure show the deep branch of the ulnar nerve (more radial) and the
superficial branch of the ulnar nerve within Guyon’s canal. Guyon’s canal is
approximately 4 cm long beginning at the proximal extent of the transverse carpal
ligament and ends at the aponeurotic arch of the hypothenar muscles. Many structures
comprise the boundaries of Guyon’s canal. The floor, for example, consists of the
transverse carpal ligament, the pisohamate and pisometacarpal ligaments, and the
opponens digiti minimi. Within Guyon’s canal, the ulnar nerve bifurcates into the
superficial and deep branches. The ulnar artery is immediately adjacent and radial to
the ulnar nerve. The median nerve is visualized within the carpal tunnel, and the
palmar cutaneous branch is more radial to Guyon’s canal and volar to the carpal
tunnel. The radial and ulnar digital nerves to the little finger are branches off of the
superficial branch of the ulnar nerve distal to its emergence from Guyon’s canal. The
ulnar artery is the round structure located radial to the branches of the ulnar nerve
within Guyon’s canal. Adjacent to the ulnar artery are two small veins. The dorsal
cutaneous branch of the ulnar nerve branches from the ulnar nerve in the distal
forearm, well proximal to Guyon’s canal. The common digital artery to the fourth
web branches from the superficial palmar arch distal to Guyon’s canal. The hook of
the hamate is clearly seen in the figure, orienting the observer to the ulnar side of the
wrist

(SAE08OS.30) A patient has carpal tunnel syndrome that requires surgical release.
The patient wants to know about the risks and benefits of the various surgical
techniques for carpal tunnel release because they have read that an endoscopic
technique is the best. What is the most significant advantage of endoscopic release
over traditional open release? Review Topic

1 Decreased rate of nerve complications

2 Decreased postoperative pain and analgesic use

3 Decreased rate of recurrence of carpal tunnel symptoms

4 Lower rate of vascular complications

5 Lower infection rate

PREFERRED RESPONSE 2

Carpal tunnel release is usually performed in one of two ways: open or


endoscopically. There has always been great controversy and differing opinions about
the advantages and disadvantages of either technique. Recent prospective randomized
trials have shown that the primary advantage of the endoscopic technique is decreased
pain in the postoperative period. A lower rate of vascular complications was not
observed in these most recent studies. Likewise, nerve injury and infection rates were
not different in these studies.

(SAE11OS.180) A 54-year-old woman who has a history of undergoing left


trapezium excision with ligament reconstruction and tendon interposition using the
entire flexor carpi radialis performed by another surgeon, now reports left basilar
thumb pain. Examination reveals pain and subluxation of the carpometacarpal joint
with axial loading. The metacarpophalangeal joint hyperextends to 60 degrees, but
radiographs show intact joint space. What is the best option to improve function?
Review Topic

1 Bracing with a hand-based thumb spica splint

2 Pinning of the carpometacarpal joint

3 Pinning of the carpometacarpal and metacarpophalangeal joints

4 Carpometacarpal revision stabilization

5 Carpometacarpal revision stabilization and metacarpophalangeal joint fusion

PREFERRED RESPONSE 5

The patient previously underwent ligament reconstruction and tendon interposition.


However, the previous surgeon failed to address metacarpophalangeal joint
hyperextension, which leads to adduction contracture and collapse of the basilar joint.
With the basilar joint causing pain and instability, repeat ligament reconstruction
should be performed. Splinting alone is unlikely to resolve instability problems.
Because the flexor carpi radialis was used, the next option is to use the abductor
pollicis longus. Additionally, the severe metacarpophalangeal joint hyperextension
should be corrected by fusion. Simple pinning is unlikely to provide long-term
stability when this degree of hyperextension exists.

(SAE08UE.31) A patient sustained a sharp laceration to the base of his left,


nondominant thumb 4 months ago. Examination reveals no active flexion but full
passive motion of the interphalangeal joint. What is the best treatment option? Review
Topic

1 Interphalangeal joint fusion

2 Intercalary tendon graft

3 Silicone rod placement

4 Primary flexor pollicis longus repair

5 Flexor digitorum superficialis transfer


PREFERRED RESPONSE 5

The patient has a chronic flexor tendon laceration. There are options to restore motion
and strength; therefore, fusion is not necessary. Full range of motion is present so the
soft tissues are suitable for a tendon transfer. A transfer of the flexor digitorum
superficialis of the ring finger to the insertion of the flexor pollicis longus on the
distal phalanx provides good results with a one-stage operation.

(SAE11OS.82) A 20-year-old woman sustained a laceration to her volar forearm 4 cm


proximal to the wrist flexion crease. She has numbness in the thumb, index, and
middle fingers. After microscopic repair of the median nerve, 2 weeks of splinting,
and commencement of a hand therapy program, the patient is most likely to require
what secondary operation 6 months after the injury? Review Topic

1 Tenolysis of the profundus tendons at the wrist

2 Nerve transfer of the ulnar motor nerve to the median motor nerve

3 Opponensplasty with the extensor indicis

4 open carpal tunnel release

5 Transfer of the extensor digiti minimi to the first dorsal interosseous tendon

PREFERRED RESPONSE 3

The patient sustained a laceration of the median nerve in what would be considered a
low median nerve injury. Standard treatment entails exploration and microscopic
repair of the median nerve. With a good quality nerve repair in a young adult, return
of some sensory function (albeit reduced compared with the normal nerve) is usual.
Return of motor function to the thenar muscles is more unpredictable. If the patient
begins a therapy program within a few weeks after nerve repair, it is unlikely that
tenolysis of the profundus tendons would be required. An open carpal tunnel release
would be unlikely to change functional return. The patient would not be expected to
have lost first dorsal interosseous function after a median nerve laceration because
this muscle is innervated by the ulnar nerve. A neurotization procedure for low
median nerve palsy has been described, but it consists of transfer of the distal anterior
interosseous nerve into the median nerve motor fascicles, not transfer of the ulnar
nerve. Therefore, the most likely secondary procedure required in this scenario is an
opponensplasty procedure to improve thumb opposition.
(SAE08OS.60) A 62-year-old woman has long-standing left thumb pain and
weakness when grasping objects. Examination reveals a positive grind test and
dynamic thumb metacarpophalangeal hyperextension. Prolonged bracing, anti-
inflammatories and oral analgesics has failed to relieve her symptoms. Clinical and
radiographic images are seen in Figures A and B. What would be the best treatment
option to relieve this patients pain and improve function? Review Topic

1 Extension osteotomy of the first metacarpal


2 Carpometacarpal joint resection arthroplasty

3 Carpometacarpal joint resection arthroplasty and metacarpophalangeal joint arthrodesis

4 Carpometacarpal joint arthrodesis and metacarpophalangeal joint arthrodesis

5 Carpometacarpal joint arthroscopy and debridement +/- ligament reconstruction

PREFERRED RESPONSE 3

This patient has end stage basilar thumb arthritis with thumb metacarpophalangeal
hyperextension of 40 degrees. She has failed conservative management. The best
treatment would be carpometacarpal joint resection arthroplasty and
metacarpophalangeal joint arthrodesis.

The mainstay of treatment for end-stage thumb carpometacarpal arthritis is excision


of the trapezium, with or without ligament reconstruction and tendon interposition.
However, if the patient has thumb metacarpophalangeal hyperextension > 30 degrees,
concurrent metacarpophalangeal joint arthrodesis is recommend to avoid adduction
contracture and poor outcomes.

Armbruster et al. reviewed the treatment of MCP joint hyperextension with CMC
arthroplasty. They recommend the following treatment algorithm:
0° to 10°= Surgical intervention is not necessary when MCP hyperextension is less
than 10°.
10° to 20°= Percutaneous pinning of the MCP joint in 25° to 35° of flexion for 3-4
weeks may be performed independently or as an adjunct to EPB transfer.
20° to 40°= Capsulodesis of the volar aspect of the MCP joint is recommened to
provide a check rein for hyperextension and Sesamoidesis has also been investigated
as an adjunctive procedure.

Mouton et al. determined if a hypermobile metacarpophalangeal joint was a causative


factor in the development of primary osteoarthritis at the base of the thumb. Using
twenty fresh-frozen cadaveric forearm specimens they recorded the motion at the
metacarpophalangeal joint in flexion neutral and hyper-extension. They showed that
flexion>extension produce the most palmar surface pressure of the trapeziometacarpal
joint which may contribute to osteoarthritis.

Figure A and B are clinical photos showing significant metacarpophalangeal joint


hyperextension. Figure C is a Ap radiograph showing advanced arthritis of the CMC
and MCP joints.

Incorrect Answers:
Answer 1: This is best does for stage I or II disease to unload the solar arthritis. This
is typically not appropriate for stage IV disease.
Answer 2: If the patient has thumb metacarpophalangeal hyperextension > 30 degrees,
concurrent metacarpophalangeal joint arthrodesis is recommend.
Answer 4: Carpometacarpal joint arthrodesis and metacarpophalangeal joint
arthrodesis would not be recommended. CMC arthrodesis is recommended in young
male heavy laborers.
Answer 5: Carpometacarpal joint arthroscopy and debridement +/- ligament
reconstruction is recommended in early stage disease.

(SAE08AN.79) Ganglion cysts about the wrist most commonly arise from what
structure? Review Topic

1 First carpometacarpal joint

2 Second carpometacarpal joint

3 Scapholunate interosseous ligament

4 Radioscaphocapitate ligament

5 Capitohamate interosseous ligament

PREFERRED RESPONSE 3

Ganglion cysts are the most common mass or mass-like lesions seen in the hand and
wrist. They arise in a variety of locations, including synovial joints or tendon sheaths.
The most common location is the dorsal/radial wrist arising from the dorsal
scapholunate interosseous ligament.
(SAE08OS.186) In obtaining informed consent for a percutaneous trigger finger
release of the thumb, what structure is discussed as being most vulnerable to injury?
Review Topic

1 Radial digital sensory nerve to the index finger

2 Radial digital sensory nerve to the thumb

3 Median motor branch to the thenar muscles

4 Volar plate of the thumb metacarpophalangeal joint

5 Ulnar digital sensory nerve to the thumb

PREFERRED RESPONSE 2

Percutaneous release of the A1 pulley for trigger finger release has become
increasingly popular. This procedure in the index, middle, and ring fingers can be
carried out with little risk to adjacent neurovascular structures. However,
percutaneous release in the thumb and little finger is controversial due to the close
proximity of the digital nerves to the A1 pulley being released. It is generally
recommended that percutaneous trigger release in the thumb and little finger be
avoided. A cadaveric study by Bain and associates showed that the average distance
of the digital nerves in the thumb from the pulley release point was 2.9 mm. The
radial digital nerve was within 2 mm of the release point in 5 of 10 hands studied.
Thus the margin of error is very small. The site of potential crossing of the digital
nerves to the little finger was variable, increasing the risk of injury as well.

(SAE08OS.159) A 15-year-old boy sustains an injury to his index finger when a


basketball impacts the tip of the finger. He reports immediate pain and swelling. A
radiograph is shown in Figure 56. Treatment should consist of Review Topic

1 buddy taping to the middle finger.


2 extension block splinting of the proximal interphalangeal joint.

3 full-time extension splinting of the distal interphalangeal joint.

4 closed reduction of the displaced fragment and percutaneous pinning.

5 open reduction and internal fixation of the fracture.


PREFERRED RESPONSE 3

The clinical history and radiographic findings are compatible with a Mallet fracture
type of injury. When the fragment is displaced but the joint remains reduced, closed
extension splinting remains the treatment of choice. Surgical treatment options should
be reserved only for cases where volar subluxation of the intact distal phalanx has
occurred because a number of studies have shown increased complication rates with
surgical treatment.

(SAE09TR.7) A 19-year-old college student reports a 1-week history of wrist pain


following an intramural rugby match. A PA radiograph is shown in Figure 6. He
denies any prior wrist injury. What is the best course of action? Review Topic

1 Closed reduction and long arm cast immobilization


2 Closed reduction and short arm cast immobilization

3 Closed reduction and percutaneous pinning

4 Open reduction and internal fixation with Kirschner wires

5 Open reduction and internal fixation with a headless, cannulated compression screw

PREFERRED RESPONSE 5

The patient has a scaphoid fracture involving the proximal pole. Surgical treatment is
recommended for such fractures because of the prolonged period of cast
immobilization necessary and the increased risk of delayed union, nonunion, and/or
osteonecrosis with nonsurgical management. A cannulated compression screw,
inserted in the central scaphoid via a dorsal approach, is biomechanically
advantageous and provides greater stability for fracture healing than Kirschner wires.
Recently, good outcomes have been reported with arthroscopic-assisted percutaneous
fixation of nondisplaced or minimally displaced scaphoid fractures.
(OBQ15.88) A 31-year-old man transects both flexor tendons to each of his index and
middle fingers while slicing vegetables. Wide-awake tumescent local anesthesia is
employed for flexor tendon repair. What is one of the primary goals of this technique?
Review Topic

1 Reduce tourniquet pain

2 Active testing of surgical repair

3 Detect bowstringing

4 Assess neurovascular bundle integrity

5 Compare 2-point sensibility with adjacent digits

PREFERRED RESPONSE 2

The primary advantage is the ability to test the repair with full active flexion and
extension by a comfortable, cooperative, unsedated, tourniquet-free patient.

Other advantages of wide-awake flexor tendon repair include the ability to (1)
evaluate if repairs will glide through pulleys, permitting the potential release of the
entire A4 and venting of half the A2 pulley if indicated, (2) repair tendons inside
tendon sheaths while demonstrating that the sheath has not been inadvertently caught,
(3) confidently initiate early active motion if the patient can make a full fist during
surgery (in contrast to passive or place-and-hold protocols).

Lalonde reviewed wide-awake hand surgery. He states that a tourniquet is no longer


required because of epinephrine hemostasis/tumescent local anesthesia, which is safe
in the digit, and its vasoconstriction is reversible with phentolamine. Sedation is
unnecessary because no tourniquet is used. Current literature suggests that Wide-
awake flexor tendon repair decreases tenolysis and rupture rates. One study reported
that maximal vasoconstriction occurs at 26min after injection.

Lalonde and Martin reviewed wide-awake flexor tendon repair and early mobilization
for zones 1 and 2. They state that historic finger necrosis was a result of of procaine,
not epinephrine. The alpha blocker phentolamine is the antidote for vasoconstriction
in the finger.

Illustration A shows methods to decrease pain with local anesthetic injection.

Incorrect Answers:
Answer 1: No tourniquet is used in this technique.
Answer 3: While bowstringing will be visible (if you cut the A2 and/or A4 pulleys), it
is not one of the primary goals of this technique. The goal is to assess repair site
excursion through pulleys and release or debulk the repair if necessary prior to skin
closure, and thus reduce the chance of the need for late tenolysis.
Answer 4: The neurovascular bundles can be assessed with the patient awake or
asleep.
Answer 5: Under local anesthesia, the infiltrated digit will be anesthetized. Testing of
2 point discrimination is not a goal of this technique.

(SAE11OS.173) A 46-year-old man sustains an injury to his left index finger while
cleaning his paint gun with paint thinner. Examination reveals a small puncture
wound at the pulp. The finger is swollen. What is the next most appropriate step in
management? Review Topic

1 Elevation and observation

2 Surgical debridement and lavage

3 Infiltration with corticosteroids

4 Infiltration with a neutralizing agent

5 Administration of antibiotics

PREFERRED RESPONSE 2

High-pressure injection injuries are associated with a high risk of amputation. The
risk of amputation is highest with organic solvents. The presence of infection and the
use of steroids do not impact the amputation rate. Amputation risk is lower if surgical
debridement is performed within 6 hours. Elevation and observation would delay
necessary care. Neutralizing agents may be used in specific situations, such as
hydrofluoric acid exposure or chemotherapeutic agent extravasation, but in high
pressure paint thinner injection, the best outcome is achieved through early surgical
lavage.

(SAE11UE.109) A patient has a mass at the base of the middle finger just distal to the
distal palmar flexion crease. The mass is 2 mm in size, firm, round, and does not
move with finger motion. It is painful with gripping activites such as a steering wheel.
What is the most appropriate management? Review Topic

1 Diagnostic ultrasound

2 MRI

3 Needle aspiration

4 Observation

5 Surgical excision
PREFERRED RESPONSE 3

The clinical scenario is of an A2 retinacular cyst. These are firm round cysts arising
from the pulley system so they do not move with tendon motion. Needle aspiration in
the office is highly effective, thus surgery can be avoided. Based on the clinical
diagnosis, ultrasound and MRI are unnecessary. Because the patient has pain and
functional limitations, observation is not recommended.

(SAE11OS.134) A 47-year-old woman sustained a nondisplaced distal radius fracture


6 months ago and is unable to extend her thumb. When performing reconstruction
using the extensor indicis proprius to the extensor pollicis longus transfer, tension is
ideally determined by securing the tendons in what manner? Review Topic

1 In maximum tension with the wrist and thumb in extension

2 In maximum tension with the wrist and thumb in neutral

3 In maximum tension with the wrist and thumb in flexion

4 According to the tenodesis effect with wrist flexion and extension

5 According to functional testing with the patient awake under local anesthesia

PREFERRED RESPONSE 5

Extensor pollicis longus rupture can result from distal radius fractures. Synergistic
tendon transfer can be achieved using the extensor pollicis longus as the motor donor.
Whereas different schemes for achieving optimal tension are available, the most
reliable method is to tension the repair under local anesthesia while asking the patient
to perform thumb flexion and extension. Tendon transfer tension can be adjusted
accordingly to achieve maximum extension without compromising active flexion
range. Other methods of tensioning are estimates at best, and maximum tensioning in
patients without neuromuscular disease is rarely used in tendon transfers.
(SAE08AN.72) Which of the following describes the correct proximal to distal
progression of the annular and cruciform pulleys of the digits? Review Topic

1 A1, C1, A2, C2, A3, A4, C3

2 A1, A2, A3, C1, C2, C3, A4

3 A1, C1, C2, A2, A3, A4, C3

4 A1, A2, C1, A3, C2, A4, C3

5 A1, A2, A3, A4, C1, C2, C3

PREFERRED RESPONSE 4

The correct progression of the annular and cruciform pulley in the digits is A1, A2,
C1, A3, C2, A4, C3. The two cruciform pulleys are collapsible elements adjacent to
the more rigid annular pulleys of the flexor tendon sheath. This arrangement enables
unrestricted flexion of the proximal interphalangeal joint.

(SAE09TR.85) A 38-year-old man caught his index finger in a volleyball net. He


noted an angular deformity of the finger that was reduced when a teammate pulled on
his finger. Three weeks later, he now reports trouble extending his finger. A clinical
photograph is shown in Figure 55. What anatomic structure is most likely injured?
Review Topic

1 Spiral oblique retinacular ligament


2 Sagittal bands

3 Volar plate

4 Central slip of the extensor tendon

5 Terminal extensor tendon


PREFERRED RESPONSE 4

The clinical photograph shows a classic boutonniere deformity. It is likely that the
patient sustained a volar dislocation of the proximal interphalangeal joint, with a
concomitant rupture of the central slip insertion of the extensor tendon.

(OBQ13.133) A 48-year-old hairdresser presents with pain and swelling of his ring
finger for 4 days. On examination, there is generalized tenderness along the entire
digit. Passive extension of the digit triggers excruciating pain. The clinical appearance
of the digit is shown in Figure A. What is the most appropriate next step in
management? Review Topic

1 Acyclovir
2 Intravenous antibiotics, splinting and elevation

3 Closed tendon sheath irrigation from the level of the A1 pulley (proximal) to the distal
interphalangeal joint (distal)

4 Continuous closed tendon sheath irrigation from the wrist (proximal) to the distal
interphalangeal joint (distal)

5 Open irrigation and debridement

PREFERRED RESPONSE 5

This patient has advanced pyogenic flexor tenosynovitis (PFT) with visible
ischemia/necrosis. Open irrigation and debridement is necessary.

Pyogenic flexor tenosynovitis is usually caused by a puncture wound (although it may


infrequently arise from hematogenous spread). The most common organism is
Staphylococcus aureus. Kanavel signs help differentiate this disease from herpetic
whitlow, septic arthritis, gout/pseudogout, and other hand infections such as
paronychia, felons, cellulitis, and deep space infections.

Draeger et al. reviewed the treatment of pyogenic flexor tenosynovitis (PFT). They
recommend open irrigation and debridement for advanced PFT and atypical or
chronic tenosynovial infections where tenosynovectomy may be indicated. Both
midaxial and volar zigzag incisions can be used.

Pang et al. reviewed factors affecting the prognosis of PFT. Of the 4 Kanavel signs,
they found that fusiform swelling was most often present (97% of patients), followed
by pain on passive extension (72%), semiflexed digit posture (69%), and tenderness
along the flexor tendon sheath (64%).

Figure A shows advanced PFT demonstrating subcutaneous purulence and local


ischemia in addition to fusiform digital swelling. Illustration A shows the Nevasier
technique of closed tendon sheath irrigation. Illustration B shows the setup for
continuous tendon sheath irrigation using nested catheters. Illustration C shows the
incision for open irrigation and debridement.

Incorrect Answers:
Answer 1: Herpetic whitlow is treated with acyclovir. Pain is at the distal tip of the
digit. Painful, clear, fluid-filled coalescent vesicles and bullae are present.
Answer 2: Nonsurgical treatment may be appropriate for patients with PFT who
present early (<48h after penetrating trauma). Recommended antibiotics include
vancomycin and piperacillin/tazobactam.
Answers 3, 4: In advanced PFT, open drainage is necessary. Closed drainage is
possible for early presenting disease. The catheter is passed from proximal to the A1
pulley to the level of the distal interphalangeal joint. In the thumb, the catheter exits
just distal to the carpal tunnel. In the small finger, if the ulnar bursa is involved, a
second catheter is placed from the A1 pulley to the wrist.

(SAE08UE.65) In surgically treating hand and finger infections in patients with


diabetes mellitus, what factor is associated with higher amputation rates? Review
Topic

1 Insulin dependence

2 Gram-positive organisms

3 Renal failure

4 Retinopathy

5 Peripheral neuropathy
PREFERRED RESPONSE 3

Patients with diabetes mellitus are prone to infection, and surgical treatment of their
infections frequently requires multiple procedures. The triad of poor wound healing,
chronic neuropathy, and vascular disease contributes to the increased infection rate.
Studies have demonstrated increased amputation rates in patients with diabetes
mellitus who have renal failure or deep polymicrobial or gram-negative infections.

(SAE11UE.69) Figures 69a and 69b show the radiographs of a 62-year-old man with
severe radially sided wrist pain. Management has consisted of wrist splinting,
nonsteroidal anti-inflammatory drugs, and activity modification, but he continues to
have pain and reports difficulty sleeping. What is the most appropriate treatment for
this patient? Review Topic

1 Arthroscopic debridement
2 Open reduction and internal fixation

3 Scaphoid nonvascularized bone graft and screw fixation

4 Scaphoid vascularized bone graft and screw fixation

5 Scaphoid excision and 4-corner fusion

PREFERRED RESPONSE 5

Scaphoidectomy and 4-bone fusion is the most appropriate management based on the
choices available. The patient has arthritic changes of SNAC (scaphoid nonunion
advanced collapse) wrist, stage III. Stage I is at the radial styloid, stage II is at the
radioscaphoid joint, and stage III is at the midcarpal joint. Arthroscopic debridement
is not appropriate in patients with arthrosis. Attempting to achieve scaphoid union is
only appropriate if there is no arthrosis or the changes are classified as stage I where
radial styloidectomy can be performed.
(SAE11UE.78) Figure 78 shows the clinical photograph of a patient who injured his
finger while playing football. He cannot actively flex the distal interphalangeal joint
of the ring finger. Which of the following is the most accurate statement regarding the
injury shown? Review Topic

1 The tendon is attached to the avulsed fragment from the distal phalanx.
2 There is no difference in time sensitivity in an acute injury whether or not the tendon has
retracted into the palm.

3 In a chronic (> 3 months) case of flexor digitorum profundus (FDP) avulsion, the FDP should
be tenodesed to the flexor digitorum sublimis (FDS).

4 If the FDP is advanced more than 1.5 cm, there is a risk for quadriga effect.

5 The method of repair does not affect repair gapping or strength of the tendon repair.

PREFERRED RESPONSE 4

Overadvancement of the FDP tendon is one of the causes of the quadriga effect.
Relative shortening of an FDP tendon decreases the excursion of the neighboring FDP
tendons because they originate from a common muscle belly. The patient reports a
weak grasp. Answer 1 is not correct because there can be a fracture and the tendon
can avulse off of the fracture fragment (Trumble JHS-A 1992). Whether the tendon
has retracted into the palm or not does matter because retraction into the palm allows
pulleys to collapse and contract and it also means that the vinculae have been stripped
off of the tendon. Regarding answer 3, in chronic cases where the FDS is intact and
strong, many patients may be better off with a sublimis finger and no FDP
reconstruction that could, in the worst case scenario, worsen a functional proximal
interphalangeal joint. Regarding the repair method, there is recent research showing
method of repair (button vs anchor), suture type, and method do affect the
biomechanical properties of the repair.
(OBQ13.103) A 26-year-old man presents with chronic hand weakness. The clinical
appearance of his hand, and radiographs are shown in Figures A through C. Surgical
exploration and decompression is performed. Besides addressing thumb
interphalangeal and index distal interphalangeal joint flexion, which is the most
appropriate treatment to restore thumb opposition? Review Topic

1 Ring flexor digitorum superficialis transfer to the abductor pollicis brevis


2 Extensor indicis proprius transfer to the abductor pollicis brevis

3 Neurotization of thenar muscles

4 Camitz palmaris longus transfer to the abductor pollicis brevis

5 Thumb carpometacarpal joint arthrodesis

PREFERRED RESPONSE 2

This patient has a high median nerve neuropathy because of a supracondylar spur and
ligament of Struthers. Reconstruction is best performed with extensor indicis proprius
(EIP) transfer to the abductor pollicis brevis (APB).

In low median nerve palsy, the primary concern is restoration of thumb opposition. In
high median nerve palsy, thumb opposition and IP flexion, and index and middle
finger flexion have to be addressed. The four common opposition transfers include (1)
ring or long FDS, (2) EIP, (3) Camitz palmaris longus (PL), or the Huber abductor
digiti minimi (ADM).

Anderson et al. reviewed EIP transfer vs FDS transfer. They found a higher
percentage of excellent results in the EIP group. In their series, complications
included index finger extensor lag (EIP transfer if the extensor expansion was not
repaired) and limited donor finger extension because of lateral band damage or
adhesions between the remaining FDS tendon and flexor sheath (FDS transfer).

Cawrse et al. modified the Huber ADM opponens transfer by releasing the proximal
end to prevent compression of the ulnar nerve in Guyon's canal by the rotated ADM
belly. They found that this technique successfully restored opposition and thenar bulk.

Figure A shows thenar wasting. Figures B and C show a supracondylar spur. The
ligament of Struthers attaches from this spur to the medial epicondyle, under which
median nerve and brachial artery pass. Illustration A shows EIP transfer. Illustration B
shows FDS transfer. Illustration C shows Camitz PL transfer. Illustration D shows
Huber ADM transfer.

Incorrect Answers:
Answer 1. The FDS is denervated in a high median nerve palsy.
Answer 3: Neurotization is performed for pre-ganglionic brachial plexus injuries.
Answer 4: The palmaris longus is denervated in a high median nerve palsy. A
palmaris longus transfer does not restore thumb pronation and metacarpal flexion,
thus not simulating opposition.
Answer 5: Thumb CMC arthrodesis will not restore opposition.

(SAE07PE.82) A 4-year-old child was born with bilateral congenital radial clubhands.
Which of the following associated conditions is a contraindication to centralization of
the hands on the ulna? Review Topic

1 Congenital scoliosis

2 Hypoplastic thumb

3 Tracheoesophageal fistula

4 Imperforate anus

5 Lack of elbow flexion

PREFERRED RESPONSE 5

Patients born with bilateral radial clubhands may have difficulty getting their hands to
their mouth. The centralization procedure would take away that ability if there is a
lack of elbow flexion.
(SAE08OS.109) A 35-year-old man sustained the injury shown in Figure 37. Three
months after closed management, the patient reports the sudden inability to extend his
thumb. What is the next most appropriate step in management? Review Topic

1 Observation
2 Primary repair of the extensor pollicis longus

3 Primary repair of the extensor pollicis brevis

4 Transfer of the extensor indicis proprius to the extensor pollicis longus

5 Transfer of the extensor indicis proprius to the extensor pollicis brevis

PREFERRED RESPONSE 4

Extensor pollicis longus rupture after nondisplaced or minimally displaced distal


radius fractures occurs due to nutritional compromise and tendon attrition. Primary
repair is almost never possible, and tendon transfer using the extensor indicis
proprius, or tendon grafting, is indicated.

(SAE08UE.47) A 17-year-old high school football player reports wrist pain after
being tackled. Radiographs are shown in Figures 22a through 22c. What is the
recommended intervention? Review Topic
1 Pedicled vascularized bone graft
2 Long arm thumb spica cast

3 Percutaneous screw fixation

4 Corticocancellous bone grafting via a volar approach (Matti-Russe)

5 Open reduction and differential pitch screw placement via a dorsal approach

PREFERRED RESPONSE 5

The patient has an acute fracture of the proximal pole. A 100% healing rate has been
reported for open reduction and internal fixation of proximal pole fractures via a
dorsal approach. This allows for direct viewing of the fracture line, facilitates
reduction, and bone grafting can be done through the same incision if necessary. A
vascularized or corticocancellous graft is reserved for nonunions. Proximal fractures
are very slow to heal with a cast, if they heal at all. As a small fragment, percutaneous
fixation is very difficult and has been reported for waist fractures.

(OBQ15.252) A healthy 12-year-old girl presents with right elbow pain after a fall off
her bicycle 3-days ago. She has full, painless flexion and extension of the elbow with
30-degrees of fixed forearm pronation. Radiographs are shown in Figures A and B.
Which of the following statements is true regarding this patient's condition? Review
Topic

1 Caused by trauma
2 Caused by failure of formation

3 Caused by failure of segmentation

4 Associated with transverse deficiency of the forearm

5 Always requires surgical intervention with osteotomy and fusion


PREFERRED RESPONSE 3

The patient has congenital proximal radioulnar synostosis (RUS), which is caused by
a failure of segmentation.

The proximal ends of the radius and ulna are temporarily united during elbow
formation and share a common perichondrium. Congenital radioulnar synostosis is a
primary developmental anomaly of radioulnar differentiation and segmentation in the
7th week of gestation. Familial cases of RUS tend to follow autosomal dominant
inheritance and is bilateral in approximately 60% of cases. RUS has also been found
to be associated with chromosomal abnormalities, most commonly sex chromosome
duplications. The majority of patients can be treated nonoperatively. Patients with
bilateral RUS and those with a pronation deformity of >60 degrees may be candidates
for osteotomy and fusion.

Elliot et al performed a retrospective cohort study evaluating patients with proximal


radioulnar fusion and posterior radial head dislocations. The authors found suggestion
of a developmental relationship between posterior dislocation of the radial head and
proximal radioulnar fusion as supported by the fact that both anomalies were found to
occur in the same patient. The anomalies are related primary developmental
anomalies of radioulnar differentiation and segmentation.

Cleary et al evaluated the natural history and functional status of 23 patients with
congenital proximal radio-ulnar synostoses. The forearms were fixed at an average of
30 degrees of pronation and the position of the forearm was not related to subjective
functional limitations, employment status or result of objective tests. The authors
concluded that operative treatment is rarely indicated and more emphasis should be
placed on objective functional tests rather than the position of the forearm.

Figures A and B are AP and Lateral views of the right elbow demonstrating proximal
radioulnar synostosis.

Incorrect Answers:
Answer 1: Although proximal radio-ulnar synostosis can be post-traumatic, the
patient reports an acute history of trauma, which is unlikely to result in her
presentation of synostosis.
Answer 2: RUS is a result of failure of segmentation.
Answer 4: RUS is associated with posterior dislocation of the radial head. Transverse
deficiency of the forearm is a distinct entity.
Answer 5: RUS rarely requires surgical intervention.
(SAE08AN.32) A patient undergoes the procedure shown in Figure 19. An important
part of this procedure is preservation of what wrist ligament? Review Topic

1 Radioscaphocapitate
2 Scapholunate interosseous

3 Ulnotriquetral

4 Volar radioulnar

5 Deep proximal capitohamate

PREFERRED RESPONSE 1

Proximal row carpectomy is a salvage wrist procedure that yields a surprisingly stable
construct. This has been attributed to two factors: 1) the congruency of the head of the
capitate in the lunate fossa (this articulation is less congruent than the native
lunate/lunate fossa relationship, but surprisingly stable), and 2) preservation of the
radioscaphocapitate ligament, the most radial of the palmar extrinsic ligaments, which
prevents ulnar subluxation after proximal row carpectomy.

(OBQ15.58) A 40-year-old homeless man comes to the emergency room after his tent
got blown away by the wind. He has pain and discoloration in his hands, shown in
Figures A and B. The weather is -15°C (5°F) with wind speed of 25km/h (15.5mph).
When is the most appropriate time to perform surgical debridement/amputation?
Review Topic
1 After immersion in a 50°C water bath for 30min, followed by re-evaluation after 6 hours
2 After immersion in a 42°C water bath for 30min, followed by re-evaluation after 24 hours

3 After demarcation, which may take 1 to 3 days

4 After demarcation, which may take 1 to 3 weeks

5 After demarcation, which may take 1 to 3 months

PREFERRED RESPONSE 5

Demarcation takes 1-3 months. Surgical intervention should occur after that time
period.

Frostbite occurs more commonly in males (10:1) in the 30-50 year old age group.
Homeless people and outdoor workers are most at risk. About 20% of injuries involve
the upper limb, 47% involve the lower limb, 30% involve both upper and lower limbs
and 3% involve the head and face. The most important host risk is alcohol abuse. The
risk of frostbite is low when air temperatures are above 14°F (–10°C), regardless of
wind velocity, but is high at temperatures below –13°F (–25°C) even when there is
little or no wind.

Golant et al. reviewed cold exposure injuries. They state that the spectrum of injury
includes nonfreezing injuries (chilblain and trench foot), as well as freezing injuries
(frostnip or frostbite). Rewarming should not begin until definitive medical care can
be provided to avoid freeze-thaw cycles. Rewarming should be rapid and for an
affected limb should be performed by submersion in warm water at 104° to 107.6°F
(40° to 42°C) for 15 to 30 minutes. Debridement is delayed until there is a clear
demarcation, a process that takes from 1 to 3 months.

Figures A and B show the initial appearance of frostbite after 12 hours of exposure.
Illustrations A, B and C show the appearance at 1 week, 6 weeks and after
amputation, respectively. Illustration D shows the different levels of tissue damage
after cold exposure. Illustration D shows the treatment for frostbite.

Incorrect Answers
Answers 1 and 2: Rapid rewarming should be in warm water (40° to 42°C).
Rewarming at a lower temperature reduces the likelihood of tissue survival.
Rewarming at a higher temperature may cause thermal burns and worsen the injury.
Debridement occurs at 1-3 months, after demarcation.
Answers 3 and 4: Debridement occurs at 1-3 months, after demarcation.

(OBQ14.256) A 50-year-old construction laborer has pain and numbness in his


dominant right hand for the past 5 years that often wakes him up at night. He brings
wrist splints his occupational health specialist provided him, which fit him well, but
no longer alleviate his symptoms. He has a strong grip and pinch, but objective
numbness over the thumb, index, and long fingers. He has positive flexion-
compression and scratch collapse tests at the carpal tunnel only, and he has neither
pain nor a Tinnel's sign over the flexor-pronator mass. You recommend: Review
Topic

1 Guided hand therapy with an occupational therapist

2 Modified work responsibilities until symptoms completely resolve

3 Carpal tunnel release on an elective basis

4 MRI of the cervical spine and referral to a spine surgeon

5 Serial carpal tunnel injections

PREFERRED RESPONSE 3

This patient has a strong history and physical examination findings of carpal tunnel
syndrome and should be offered an elective carpal tunnel release.

Carpal tunnel classically presents with numbness and pain in the median nerve
distribution, which worsens at night. Physical exam findings such as a Tinnel's sign,
Flexion-compression testing, Phalen's test, and Scratch collapse testing over the
carpal tunnel allow practitioners to make a clinical diagnosis. Injections and
electrodiagnositic studies can assist in diagnosing carpal tunnel syndrome, but are
most helpful in cases where the diagnosis is less clear clinically. Additionally, AAOS
guidelines recommend electrodiagnostic studies if surgical intervention is planned
(Grade B evidence).

The study by Graham concluded that in patients with a strong history and clinical
examination consistent with carpal tunnel syndrome, electrodiagnostic tests were very
unlikely to change the clinical diagnosis. Furthermore, in patients with poor pre-test
probability, electrodiagnostic studies were found to be most useful in ruling out carpal
tunnel syndrome.
The chapter in OKU 10 by Rozental and LaPorte discusses the clinical presentation
and management of carpal tunnel syndrome.

The video below shows a scratch collapse test being performed on a patient with
recurrent carpal tunnel syndrome after bilateral releases.

Incorrect Answers:
Answer 1: Alternative conservative strategies are less likely to be effective in
relieving his symptoms than a carpal tunnel release
Answer 2: Long-lasting carpal tunnel syndrome is unlikely to resolve by activity
modification alone
Answer 4: MRI of the cervical spine is unlikely to change management in this case.
Answer 5: Serial carpal tunnel injections would not be appropriate in a patient that
can undergo an elective carpal tunnel release

(SAE09TR.51) A 42-year-old college professor reports persistent pain at the


hypothenar eminence 9 months after falling from his bicycle. Initial radiographs were
reportedly normal. Use of a wrist splint for the last 2 months has failed to provide
relief. A radiograph obtained by his primary care physician prior to referral is seen in
Figure 31. What is the most appropriate treatment? Review Topic

1 Continued splint immobilization for 6 weeks


2 Pisiform excision

3 Hook of hamate excision

4 Open reduction and internal fixation with bone grafting

5 Lunotriquetral arthrodesis

PREFERRED RESPONSE 2
The oblique radiograph of the wrist reveals a displaced fracture of the pisiform that
usually occurs following a direct blow or sudden contraction of the flexor carpi
ulnaris tendon. Persistent pain secondary to pisotriquetral incongruity and
degenerative arthritis warrants pisiform excision, which does not compromise wrist
flexion strength. Pisiform fractures are usually missed on routine radiographic views.
An oblique or carpal tunnel view can be helpful in visualizing the pisotriquetral joint.

(SAE11AN.22) A 20-year-old man has a dorsal metacarpophalangeal dislocation of


the index finger. Multiple attempts to reduce the dislocation in the emergency
department have not been successful. What structure is most likely preventing the
joint from being reduced? Review Topic

1 First dorsal interosseous

2 Radial collateral ligament

3 Ulnar collateral ligament

4 Natatory ligament

5 Volar plate

PREFERRED RESPONSE 5

The volar plate is the structure that usually prevents the finger metacarpophalangeal
joint from reducing. Blockage by the first dorsal interosseous is not a common reason
for an irreducible metacarpophalangeal joint dislocation. None of the other structures
commonly prevent metacarpophalangeal joint reduction.

(OBQ14.75) A 49-year-old construction worker presents with pain in his small finger
for 2 months. Figure A shows the Allen's test on the involved hand. An arteriogram is
obtained and is shown in Figure B. Following a period of rest and activity
modification, he returns 1 month later with ulceration at the tip of his small finger. At
this point, a color flow doppler ultrasound image of the ulnar artery is shown in
Figure C. The digital brachial index is 0.4. What is the next best step? Review Topic
1 Catheter-directed thrombolysis
2 Systemic thrombolysis

3 Ulnar artery stenting

4 Ulnar artery ligation

5 Ulnar artery reconstruction

PREFERRED RESPONSE 5

This patient has hypothenar hammer syndrome complicated by ulnar artery aneurysm
which eventually thrombosed, leading to digital ischemia and ulceration. Treatment
involves resection of the involved segment and reconstruction with a vein (or artery)
graft as the gap is usually too great for direct repair.

Hypothenar hammer syndrome is post-traumatic digital ischemia from thrombosis of


ulnar artery at Guyon's canal and is associated with the use of vibrating tools (e.g.
mechanics, construction workers). The hamate hook functions as an anvil, leading to
ulnar artery aneurysms, which may cause thrombosis. The digital brachial index
(DBI) is a good predictor of outcome and treatment. DBI of 0.85 to 1.25 is normal. If
the DBI > 0.7, there are minimal symptoms, the digit is viable, surgery is not
warranted. If the DBI < 0.7, arterial reconstruction is suggested. DBI < 0.5 is
associated with severe symptoms and digital gangrene.

Chloros et al. reviewed 13 patients who underwent artery excision and reversed
interpositional vein grafting. They found that 10 (77%) were patent at 2 years.
Symptoms were improved from initial presentation, and isolated cold stress test
responses were not different from normal controls. Two patients with obstructed
grafts still had pain, numbness and cold sensitivity.They concluded that successful
reconstruction improves function and quality of life.

McClinton et al. reviewed hypothenar hammer syndrome. They describe the ulnar
artery as the most common site of true and false aneurysms, where aneurysm is
defined as permanent localized vessel wall dilatation of >= 50% of normal. The cause
of ulnar aneurysms is usually hypothenar hammer syndrome. They reconstructed with
ipsilateral veins from the forearm or foot dorsum, or artery segments from the
thoracodorsal artery, the superficial inferior epigastric artery, or the descending
branch of the lateral femoral circumflex artery.

Figure A shows the results of the Allen's test. He has an incomplete palmar arch, with
ulnar border digits supplied exclusively by the ulnar artery, and radial border digits by
the radial artery. Figure B is an arteriogram showing aneurysmal dilatation of the
ulnar artery. Figure C is a longitudinal color flow doppler image showing an area of
flow (red, right) adjacent to a thrombus (no color, left). Illustration A shows reversed
saphenous vein reconstruction of the ulnar artery and extending into the superficial
arch.

Incorrect Answers:
Answer 1: Endovascular thrombolysis is an option for acute thrombosis without
aneurysmal dilatation. In the presence of an aneurysm, the involved segment has to be
excised and reconstructed to prevent recurrence and progression of
ulceration/ischemia.
Answer 2: Systemic thrombolysis is not used for this condition.
Answer 3: Stenting is an option for arterial stenosis but not for thrombosis.
Answer 4: Ulnar artery ligation is contraindicated, especially in the presence of an
incomplete palmar arch.

(OBQ14.39) A 2-year-old child presents with flexion contracture of the digit as


shown in Figure A. The triggering persists after A1 pulley release. At the revision
surgery, the following steps are necessary: Review Topic

1 Horizontal incision and complete A2 pulley release. If triggering persists, perform


circumferential debulking of the flexor digitorum profundus tendon.
2 Extensile incision and release of 1 slip of the flexor digitorum superficialis tendon. If
triggering persists, release the A3 pulley.

3 Extensile incision and release of 1 slip of the flexor digitorum superficialis tendon. If
triggering persists, perform circumferential debulking of the flexor digitorum profundus
tendon.
4 Horizontal incision and complete A3 pulley release. If triggering persists, perform
circumferential debulking of the flexor digitorum profundus tendon.

5 Horizontal incision and release of 1 slip of flexor digitorum superficialis. If triggering


persists, release the A3 pulley.

PREFERRED RESPONSE 2

An extensile incision should be chosen to approach the pediatric trigger finger as the
pathology could lie at many sites. The ulnar FDS slip should be resected. If triggering
persists, release of the A3 pulley may be performed, or release of the remaining FDS
slip.

In the pediatric population, trigger thumbs are 10 times more common than trigger
fingers. Trigger thumb presents with flexion contracture, and is caused by constriction
at the A1 pulley. Trigger fingers present with a palpable Notta's nodule proximal to
the A1 pulley, flexion contracture, and triggering. Trigger fingers are caused by an
abnormal relationship of the FDP and FDS tendons, a more proximal decussation of
the FDS, nodules in either the FDS or FDP, a thickened A-2 pulley, or a tight A-3
pulley.

Moon et al. reviewed 7700 children prospectively and found no incidence of trigger
finger or thumb. They then reviewed 35 trigger thumbs and 8 trigger fingers. Surgical
release was performed in 23 thumbs. Spontaneous resolution was noted in 12 thumb
cases and all 8 fingers. They concluded that trigger finger develops earlier in life and
has a higher rate of spontaneous resolution.

Cardon et al. retrospectively reviewed 239 trigger digits in 176 children. Trigger
fingers accounted for 14% of digits and 10% of patients. Average age at presentation
was 31 months, and age at onset was 11 months. There were 9 long, 4 ring and 5
small trigger fingers.

Figure A shows flexion contracture of the long finger of the left hand secondary to
pediatric trigger finger.

Incorrect Answers:
Answer 1: Complete A2 pulley release would lead to bowstringing. A horizontal
incision does not allow proximal and distal exploration.
Answers 3: Debulking of the FDP tendon would lead to an irregular tendon surface,
which would in turn promote tendon adhesions and scarring.
Answer 4: The first step should be release of the ulnar slip of FDS. If triggering
persists, the A3 pulley can be released if triggering is seen at this location. Debulking
of the tendon should not be performed to reduce the risk of adhesions.
Answer 5: While A3 pulley release is a correct second step if triggering persists in
spite of A1 pulley and FDS slip release, a horizontal incision will not allow distal
exploration.
(SAE09TR.32) A 19-year-old collegiate baseball player injures the ring finger on his
dominant hand while sliding headfirst into second base. He reports that he is unable to
actively flex or extend the distal interphalangeal joint of the finger. Radiographs are
shown in Figures 19a and 19b. What is the anatomic lesion leading to this injury?
Review Topic

1 Rupture of the terminal extensor tendon


2 Avulsion of the volar plate

3 Rupture of the sagittal bands

4 Rupture of the spiral oblique retinacular ligaments

5 Rupture of the profundus insertion

PREFERRED RESPONSE 5

The radiographs reveal a bony avulsion of the flexor profundus insertion (Jersey
finger). The large bony fragment classifies this as a Leddy type III injury. The bony
fragment has retracted to the level of the annular pulley (A4).

(SAE08UE.59) A 17-year-old high school football player reports wrist pain 5 months
after the conclusion of the football season. A radiograph and CT scan are shown in
Figures 29a and 29b. What is the recommended intervention? Review Topic
1 Pedicled vascularized bone graft
2 Long arm thumb spica cast

3 Percutaneous screw fixation

4 Corticocancellous bone grating via a volar approach (Matti-Russe)

5 Open reduction and differential pitch screw placement via a dorsal approach

PREFERRED RESPONSE 1

The patient has a nonunion of the proximal pole of the scaphoid. Acutely, this can be
repaired with a screw alone, but as a nonunion the proximal pole has very poor
healing potential. Vacularized bone grafts have been successful for these challenging
nonunions, particularly in adolescents. A cast can be used for nondisplaced acute
waist fractures, and corticocancellous grafts can be used for nonunions of the waist.

(OBQ15.21) A 14-month-old child is brought into your office because the mother has
noticed reduced motion in the left upper extremity. The child appears at ease, playing
quietly by herself. She abducts her shoulder to pick up building blocks on the ground.
Examination reveals normal elbow flexion and extension, but diminished supination
compared with the contralateral side. Radiographs are shown in figures A through C.
What is the next best step? Review Topic
1 CT
2 MRI

3 Arthrocentesis

4 Reduction

5 Observation

PREFERRED RESPONSE 5

This child has congenital proximal radioulnar synostosis (CPRUS). There are no
apparent fractures. Observation is all that is necessary.

CPRUS is a rare condition arising as a failure of longitudinal segmentation. The


cartilaginous anlage between the radius-ulna during the 7th week in utero leaves a
persistent bridge of tissue. The average age of diagnosis is 6 years and there is a
positive family history in 20% of cases. The average position of the forearm is 30
degrees of pronation. Surgery is rarely indicated. Surgical recommendations include
positioning the dominant forearm in 10-20° pronation and the nondominant forearm
in neutral rotation (bilateral) or 30° of pronation (unilateral cases).

Cleary and Omer reviewed the natural history of CPRUS in 23 patients (10 unilateral,
13 bilateral). All forearms had fixed PRONATION, but the position did not affect
functional limitations or employment. They found 4 radiographic patterns: (1) no
bony synostosis, radial head reduced, (2) bony synostosis, radial head reduced, (3)
long bony synostosis, radial head posteriorly dislocated, (4) radial head anteriorly
dislocated.

Kozin reviewed congenital elbow abnormalities. He states that CPRUJS is bilateral in


50%. The shoulder/wrist is able to compensate. The shoulder abducts to compensate
for deficient pronation, and adducts to compensate for deficient supination. Children
may develop excessive wrist motion. Difficulties from deficient pronation involve
keyboard/tabletop activities. Difficulties from deficient supination include eating,
washing face, catching a ball and using a soap dispenser.
Figures A through C show a CPRUS of the left upper extremity. Illustrations A
through D show the Cleary and Omer classification of Types I-IV CPRUS.

Incorrect Answers:
Answers 1 and 2: Further imaging will not yield any more useful information.
Answer 3: Joint aspiration is unnecessary
Answer 4: There is no dislocation that would necessitate joint reduction.

(SAE11OS.162) A 37-year-old woman has right-hand numbness and tingling. Based


on the history and examination, carpal tunnel syndrome is suspected, and
electrodiagnostic tests also point to the same diagnosis. The patient has worn night
splints for the last 8 weeks with continued persistent symptoms. What is the next most
appropriate step in management? Review Topic

1 Continue the night splinting for 1 additional month.

2 Continue the night splinting for 3 more months.

3 Switch to full-time splinting and reevaluate in 1 month.

4 Switch to full-time splinting for 3 more months.

5 Perform carpal tunnel release.

PREFERRED RESPONSE 5

Various nonsurgical management options exist for carpal tunnel syndrome (local and
oral steroids, splinting, and ultrasound). All effective or potentially effective
nonsurgical forms of management have measureable effects on symptoms within 2 to
7 weeks of the initiation of treatment. If a treatment is not effective within that time
frame, a different treatment option should be chosen. In this case, continued splinting
is unlikely to improve symptoms and steroid injection or surgery is indicated.

(SAE09TR.76) A still active 86-year-old pastry chef falls in her kitchen and notes
pain and deformity of her little finger. There are no open wounds. Radiographs are
shown in Figures 49a and 49b. What is the most appropriate management? Review
Topic
1 Closed reduction and splinting in metacarpophalangeal (MCP) and proximal
interphalangeal (PIP) joint extension
2 Closed reduction and splinting in MCP joint flexion and PIP joint extension

3 Reduction and percutaneous fixation

4 Open reduction and internal fixation

5 Buddy taping and early range of motion

PREFERRED RESPONSE 3

The fracture of the proximal phalanx is clearly displaced. There is slight comminution
at the area of the fracture. Closed reduction is likely to fail due to the forces of the
extensor, flexor, and intrinsic mechanisms. Percutaneous fixation, unlike open
fixation techniques, avoids likely problems with stiffness.

(SAE08OS.73) A 23-year-old man sustains a dorsal dislocation of the index finger


metacarpophalangeal (MCP) joint while skateboarding. Which of the following
anatomic structures is most likely to be interposed between the articular surfaces and
account for the irreducibility of this joint by closed methods? Review Topic

1 Volar plate

2 Flexor tendon

3 Extensor tendon

4 Lumbrical tendon

5 Radial collateral ligament


PREFERRED RESPONSE 1

The most common structure preventing reduction of a MCP dislocation is the volar
plate. It is most easily treated by a dorsal approach and a longitudinal split of the
plate. Whereas it has been described that the metacarpal head can become entrapped
between the radial lumbrical and the flexor tendon, replacement of the plate volar to
the metacarpal head during surgery allows for the joint to be easily reduced with no
attention paid to the lumbrical or flexor tendon. The collateral ligaments and the
extensor tendon have not been described as structures that prevent reduction of the
MCP joint.

(OBQ15.133) A 42-year-old woman presents with sudden left arm pain, cyanosis and
paresthesia. Examination reveals mildly diminished grip strength, intact sensation
over all digital pulps. The brachial, radial and ulnar pulses are not palpable, although
Doppler examination detects a monophasic waveform. Treatment for the most likely
diagnosis may include all of the following EXCEPT Review Topic

1 Aortic root replacement

2 Intra-arterial thrombolysis

3 Angioplasty

4 Manual aspiration

5 Stenting

PREFERRED RESPONSE 1

This patient has non-occlusive acute left subclavian artery thrombosis (absent pulses
but detectable doppler). Treatment can include percutaneous approaches (Answers 2-
5). Aortic root replacement is not indicated for subclavian arterial disease alone. It is
however indicated for ascending thoracic aortic dissection.

Acute subclavian artery thrombosis is a rare cause of upper limb ischemia. Risks
include aortic arch abnormality, aortic arch syndrome, trauma, arterial catheterization,
pre-existing atheroma/stenosis at the subclavian artery ostium. Patterns include slow
occlusion (atherosclerosis), blue finger syndrome (repeated focal microemboli) and
acute global ischemia (large emboli).The diagnosis is made by conventional
angiography, which has the added advantage of allowing percutaneous treatment.
Treatment strategies include (1) medical therapy (heparin, aspirin, clopidogrel, IV
thrombolysis, glycoprotein IIb/IIIa antagonists [abciximab, eptifibatide, tirofiban]),
(2) open embolectomy and bypass grafting, and (3) percutaneous endovascular
procedures.

Pellerin et al. describe 2 cases of percutaneous endovascular management of acute left


subclavian artery thrombosis. The 1st case was treated with thrombolysis, stenting
and manual aspiration of the thrombus. The 2nd case was treated with
anticoagulation, stenting, and long term anticoagulation.

Rapp et al. reviewed patients with proximal lesions causing upper extremity ischemia.
Fourteen had extrinsic compression, 17 had brachiocephalic atherosclerosis, 1 had an
occluded graft, 1 had Takayasu's arteritis, 1 had radiation atherosclerosis. Some
patients had ipsilateral vertebral arterial reversal of flow with neurologic symptoms
characteristic of VERTEBROBASILAR insufficiency. Patients with occlusion had
arterial reconstruction, bypass and endarterectomy. Five required amputation, 4
because of delayed diagnosis. They concluded that these lesions should not be
overlooked as consequences are devastating.

Illustration A shows acute non-occlusive left subclavian artery thrombosis. Panel A


shows non-occlusive left subclavian artery thrombosis on aortic arch angiography
(black arrow). Panel B shows the dilatation of the 2 self-expandable stents (white
arrow). There is a protection balloon (black arrow) occluding the left vertebral artery
preventing vertebral artery embolism. Panel C shows final aortic arch angiography
demonstrating the crushed thrombus between the stent and the arterial wall (black
arrow).

Incorrect Answers:
Answers 2-5: Percutaneous approaches may include thrombolysis, stenting,
percutaneous transluminal ballon angioplasty, and manual aspiration.

(SAE09TR.33) A 72-year-old man was involved in an automobile accident 4 weeks


ago. Initially he noted pain about his nondominant left shoulder, which resolved
within a few weeks after the accident. He now describes trouble with gripping and
carrying items in his left hand. Radiographs are shown in Figures 20a through 20c.
His signs and symptoms are the result of injury to which of the following ligaments?
Review Topic
1 Volar scapholunate interosseous
2 Dorsal scapholunate interosseous

3 Long radiolunate

4 Short radiolunate

5 Dorsal intercarpal

PREFERRED RESPONSE 2

The radiographs reveal a gap between the scaphoid and lunate bones, indicative of
disruption of the scapholunate ligament complex. The three components of the
complex are the dorsal scapholunate ligament, the volar (or palmar) scapholunate
ligament, and the proximal fibrocartilaginous membrane, listed in decreasing yield
strength. Disruption of the stout dorsal interosseous ligament is required for
scapholunate dissociation to occur.

(SAE08AN.78) In Dupuytren’s disease, the retrovascular cord typically displaces the


radial proper digital nerve of the ring finger in what direction? Review Topic

1 Palmarly and radially

2 Dorsally and ulnarly

3 Palmarly and ulnarly

4 Dorsally and radially

5 Directly dorsal
PREFERRED RESPONSE 3

Retrovascular cords are common in Dupuytren’s disease and commonly require


surgical treatment. Nerve injury in Dupuytren’s surgery is an infrequent complication
that occurs partly because the digital nerves can be displaced from their normal
anatomic relationships by retrovascular cords. The nerves are displaced superficially,
toward the center of the digit (palmarly and ulnarly). This displacement is typically
seen at the level of the metacarpophalangeal joint.

(SAE08UE.5) What ligament is the primary stabilizer of the wrist following a


proximal row carpectomy? Review Topic

1 Dorsal radiocarpal

2 Dorsal intercarpal

3 Radioscaphocapitate

4 Ulnocapitate

5 Ulnotriquetral

PREFERRED RESPONSE 3

The radioscaphocapitate ligament is the prime stabilizer between the radius and
capitate, preventing ulnar translocation of the carpus. Its oblique orientation prevents
the carpus from drifting ulnarly. This stout ligament must be protected when excising
the scaphoid.

(OBQ09.152) A 6-year-old boy presents to your office with a forearm deformity and
the radiographs shown in Figure A. What additional tests must be ordered for this
patient? Review Topic
1 CBC
2 CBC and renal ultrasound only

3 CBC, renal ultrasound, and echocardiogram

4 CBC and MRI

5 No additional testing needed

PREFERRED RESPONSE 3

The clinical presentation is consistent with radial clubhand or radial longitudinal


deficiency. A CBC, renal ultrasound, and echocardiogram should be obtained to
screen for associated congenital abnormalities.

Radial clubhand is commonly associated with thrombocytopenia-absent radius


(TAR), Fanconi's anemia, Holt-Oram syndrome (congenital heart disease), and
VACTERL (vertebral, anal, cardiac, tracheal, esophageal, renal, and limb) syndromes.
A CBC, renal ultrasound, and echocardiogram are the basics that must be ordered to
screen for the other associated conditions. If Fanconi's anemia is suspected,
chromosomal breakage studies using diepoxybutane or mitomycin C allow for
detection of this disorder prior to the onset of pancytopenia. It is important to obtain
complete blood counts as the hematopoietic system is most commonly involved.

Maschke et al. emphasize that all children presenting with radial longitudinal
deficiency, regardless of severity, require a renal ultrasound, echocardiogram, and
complete blood count to evaluate the potential for associated systemic conditions.
They report addition genetic testing may be warranted to rule out specific conditions,
such as Fanconi's anemia.

Goldfarb et al. reviews thumb function in these patients. They report that while the
thumb in TAR syndrome patients is of relatively normal size and shape, due to its
position of metacarpophalangeal flexion, function is significantly impaired.

Goldfarb et al. evaluated 164 patients with radial clubhand or associated congenital
defects. Of these 25 patients had thrombocytopenia absent radius syndrome; 22
patients had vertebral, anal, cardiac, tracheoesophageal, renal, and limb abnormalities
association; 7 patients had Holt-Oram syndrome; and 1 patient had Fanconi anemia.
There were 32 patients with cardiac abnormalities and 60 patients with spinal or
lower-extremity musculoskeletal abnormalities. They therefore recommend, in
addition to a CBC, renal ultrasound, and echocardiogram, that spinal radiographs be
obtained.

Figures A demonstrates the radiographs of a radial clubhand or radial longitudinal


deficiency.

(OBQ14.33) A 13-month-old male presents to clinic with a with a fixed flexion


deformity as seen in Figure A. When discussing treatment options with his parents it
should be noted that delaying surgical intervention until age 2 years will likely cause:
Review Topic

1 Permanent flexion contracture of the thumb


2 The patient to become left hand dominant

3 A slight delay in motor milestones that will catch up after surgical release

4 A slight delay in motor milestones that will not catch up after surgical release

5 No sequelae

PREFERRED RESPONSE 5

Pediatric trigger thumb can be treated non-operatively until age 2 years without
significant risk of sequelae. After age 2 years surgical intervention is recommended
because full spontaneous resolution of the contracture after age 2 years is unlikely.

Pediatric (or "congenital") trigger thumb typically progresses from notable triggering
of the thumb interphalangeal (IP) joint that progresses to a fixed contraction
deformity. Non-operative treatment may be attempted including passive extension
exercises or extension splinting, and full spontaneous resolution during the first two
years of life has been reported. After age 2 years, spontaneous resolution of the
contracture is rare, and surgical intervention is recommended. Delaying surgical
release, even in the face of a fixed contracture, until age 2 years is not associated with
known sequelae. Surgical intervention involves release of the A1 pulley of the thumb,
and leads to 95% resolution in all age groups.

Shah et al. present a review of pediatric trigger thumb and trigger fingers. They note
that non-operative treatments including passive exercises and extension splinting are
of questionable efficacy, but may be attempted and resolution of the triggering may
occur up until age 2 years. After age 2 they recommend surgical treatment since the
contracture is unlikely to fully spontaneously resolve after that point.

Baek et al. present a prospective cohort of patients with pediatric trigger thumb
treated non-operatively (and with no stretching or splinting) and followed for 48
months. They found that 60% of patients resolved spontaneously, and that those who
did not fully resolve did have improvement in their thumb IP range of motion. Most
of the improvements seen were within the the first 2 years of follow-up after initial
diagnosis (in some patients up to age 4 years).

Figure A is a child's hand with a fixed flexion deformity of the interphalangeal joint
of the thumb consistent with pediatric trigger thumb.

Incorrect answers:
Answer 1: Delaying surgical intervention of pediatric trigger thumb until age 2 years
is not known to result in any permanent flexion contracture.
Answer 2: Having a right sided trigger thumb until age 2 years is not known to
increase the liklihood of becoming left hand dominant. Handedness is not typically
developed until closer to age 4 years.
Answers 3 and 4: No delay in motor milestones is expected in relation to a pediatric
trigger thumb treated non-operatively.

(SAE11AN.65) A 34-year-old woman who underwent release of her first dorsal


compartment at the wrist for de Quervain's tenosynovitis 3 months ago continues to
report radial-sided wrist pain and tenderness similar to what she had prior to surgery.
Examination appears classic for de Quervain's with a positive Finkelstein's test and
continued pain with palpation over the first dorsal compartment. What is the likely
source of her continued pain? Review Topic

1 Tendon subluxation

2 Intersection syndrome

3 Injury to the dorsal radial sensory nerve

4 Tendon injury to the abductor pollicis longus (APL) tendon

5 Unreleased extensor pollicis brevis (EPB) tendon


PREFERRED RESPONSE 5

Persistant pain after first dorsal compartment release is often the result of failure to
release all potential septations or compartments. It has been found that 24% to 34% of
wrists have a separate compartment involving the EPB or APL. If each tendon is not
identified, an incomplete release can result, causing continued symptoms. Intersection
syndrome is more proximal, pain is not over the first dorsal compartment. Radial
sensory nerve injury would not result in a positive Finkelstein's test. Tendon
subluxation and tendon injury usually do not cause pain over the first dorsal
compartment with palpation.

(SAE08OS.63) Figures 19a and 19b show the clinical photographs of a 2-year-old
child who cannot straighten the right thumb. Examination shows active flexion of the
thumb, but the interphalangeal joint cannot be passively extended. Extension is not
improved with flexion of the wrist or metacarpophalangeal joint of the thumb. What is
the most appropriate management? Review Topic

1 Lengthening of the flexor pollicis at the wrist


2 Volar plate release and pinning of the interphalangeal joint in extension

3 Release of the A-1 pulley

4 Extension splinting of the thumb for 30 days

5 Injection of corticosteroids at the base of the thumb


PREFERRED RESPONSE 3

Congenital trigger thumbs are common and are reliably treated by A-1 pulley release.
A palpable nodule frequently is felt on the flexor tendon, confirming the diagnosis.
The interphalangeal joint does not have intrinsic contracture and extends fully after
pulley release. Corticosteroid injection is difficult and not beneficial in the child with
fixed flexion. Extension splinting has been used by some but requires several months
of splinting, not just 30 days. The flexor pollicis is of normal length and does not
require lengthening.

(SAE07SM.40) Figures 12a through 12c show the radiographs of a 28-year-old


professional baseball player who has ulnar-sided wrist pain and numbness and
tingling in the fourth and fifth digits for the past 6 weeks. Management should consist
of Review Topic

1 cast immobilization.
2 bone stimulation and splinting.

3 ulnar nerve exploration.

4 open reduction and internal fixation.

5 excision of the fragment.


PREFERRED RESPONSE 5

Hook of the hamate fractures typically occur as a result of direct force from swinging
a bat, golf club, or racket. Pain is localized to the hypothenar eminence. The injury is
best seen on a carpal tunnel view. CT will confirm the diagnosis. Chronic cases can
be associated with neuropathy of the ulnar nerve. Excision of the hook through the
fracture site usually yields satisfactory results, allowing the athlete to return to
competition.

(SAE11UE.38) Figure 38 shows the radiograph of a 41-year-old man who reports


ulnar palmar pain, decreased sensibility and tingling in the ring and little fingers, and
a grating sensation in the ulnar fingers with motion. He reports that he sustained a fall
on an outstretched hand 6 months ago. What is the most appropriate treatment option?
Review Topic

1 Ulnar gutter cast


2 Short arm cast

3 Carpal tunnel release

4 Decompression of Guyon's canal

5 Excision of a fractured hook of hamate

PREFERRED RESPONSE 5

Excision of a fractured hook of hamate is the most appropriate management. The


patient has a hook of hamate fracture with ulnar nerve compression and irritation of
the flexor tendons by the fracture surfaces; this puts the tendons at risk for rupture.
Cast treatment will most likely not gain union of the fracture and will not address the
nerve or tendon problems. Decompression of Guyon's canal alone will not address the
tendon issue.
(SAE09TR.10) A 47-year-old woman falls and sustains a direct blow to her middle
finger. She notes pain and swelling and is unable to move the proximal
interphalangeal (PIP) or distal interphalangeal (DIP) joints. Radiographs are shown in
Figures 8a through 8c. Proper management should consist of Review Topic

1 closed reduction and splinting in metacarpophalangeal (MCP) and PIP joint extension.
2 closed reduction and splinting in MCP joint flexion and PIP joint extension.

3 reduction and percutaneous intramedullary Kirschner wire fixation.

4 reduction and lag screw fixation.

5 buddy taping and early range of motion.

PREFERRED RESPONSE 4

The oblique nature of the fracture and extension of the fracture to the condyles
implies an unstable fracture. Lag screw fixation provides an excellent chance of
union, and the ability to start early range of motion.

(SBQ13PE.113) A 7-year-old nonverbal boy with severe Autism is brought to the


emergency department by his caretaker after noticing a bump over the left elbow. She
states that the patient falls often but is not sure when the bump first appeared. The
patient moves his bilateral upper extremities spontaneously and without apparent
discomfort. Examination of his left elbow is notable for a prominence over the
posterolateral elbow that is nontender. Plain radiographs are pictured in Figures A and
B. What is the next best step in management? Review Topic
1 Plain radiographs of the contralateral elbow
2 Closed reduction under sedation

3 Open reduction with annular ligament reconstruction

4 Open reduction with ulna osteotomy

5 Radial head resection

PREFERRED RESPONSE 1

The clinical picture is most consistent with a congenital radial head dislocation
(CRHD), which is often bilateral. Radiographs of the contralateral elbow can be
useful in confirming diagnosis. The management of asymptomatic CRHD is
observation.

CRHD can occur in isolation or in association with a variety of syndromes. It is most


commonly bilateral, posteriorly dislocated, atraumatic, difficult to reduce with distinct
radiographic findings that include a concave radial head and hypoplastic capitellum.
Management of asymptomatic CRHD is observation. If patients develop pain and/or
symptomatic loss of motion, radial head resection can be considered.

Manske et al reviewed the classification of congenital anomalies of the hand and


upper extremity. CRHD can occur with a number of other anomalies including
longitudinal deficiencies of the radius and proximal radioulnar synostosis. The
underlying abnormality of CRHD is thought to be failure of capitellum development,
resulting in a loss of contact pressure required for normal radial head development.
Malformation of the radiocapitellar joint leads to altered biomechanics at the proximal
radioulnar joint and abnormal ulna development.

Agnew et al performed a retrospective review of six patients with unilateral radial


head dislocations. Radiographic findings associated with congenital etiology include:
relative shortening of the ulna or lengthening of the radius, absence or hypoplasia of
the capitellum and a dome-shaped radial head with a long narrow neck. Though most
often bilateral, the authors concluded that unilateral congenital radial head dislocation
can occur.
Figures A and B are the AP and lateral radiographs, respectively, of the left elbow
showing a posterior radial head dislocation with concave radial head. Illustrations A
and B are the AP and lateral radiographs, respectively, of the right elbow in this
patient showing posterior dislocation of the radial head as well. Illustration C
demonstrates clinical and radiographic criteria for diagnosis of CRHD.

Incorrect Responses:
Answers 2, 3, 4, 5: The best management for this patient with asymptomatic CRHD is
observation and imaging of the contralateral elbow to evaluate for bilateral CRHD.

(SAE09TR.41) A 21-year-old college student fell from a balcony and landed on his
outstretched right hand. He is seen in the emergency department 4 hours later and
reports wrist pain and diffuse hand numbness. The volar forearm compartment is soft
and there is no pain with passive finger extension. Radiographs are shown in Figures
25a and 25b. Definitive treatment should consist of Review Topic

1 closed reduction and cast immobilization.


2 closed reduction, carpal tunnel release, and cast immobilization.

3 open reduction and ligament repair via a dorsal approach.

4 open reduction and ligament repair via a volar approach.

5 open reduction and ligament repair via dorsal and volar approaches and an extended
carpal tunnel release.
PREFERRED RESPONSE 5

A spectrum of perilunate injury patterns exists, with the dorsal trans-scaphoid


perilunate fracture-dislocation being the most common. Perilunate injuries are highly
unstable complex carpal disruptions that are not amenable to closed treatment. Open
reduction and internal fixation is necessary to accurately restore carpal alignment via
fracture reduction and fixation and intercarpal ligament repair. Controversy exists
regarding the need for dorsal or combined dorsal and volar approaches. Based on the
radiographic findings of a volar dislocation of the lunate and the associated median
nerve injury, the patient requires open reduction and internal fixation via combined
dorsal and volar approaches with a concomitant carpal tunnel release.

(OBQ12.235) Figure A is a radiograph of a 35-year-old women who sustained an


isolated left wrist injury after a fall onto an outstretched hand. She has been
complaining of left dorsal wrist pain since the fall. Examination reveals a positive
Watson's scaphoid shift test. What ligamentous structure is an important secondary
stabilizer to prevent dorsal intercalated segment instability (DISI) deformity in this
patient? Review Topic

1 Transverse carpal ligament


2 Dorsal intercarpal ligaments

3 Triangular fibrocartilage complex

4 Dorsal lunotriquetral ligament

5 Volar lunotriquetral ligament

PREFERRED RESPONSE 2

The integrity of the dorsal intercarpal ligaments is important in preventing dorsal


intercalated segment instability (DISI) deformity and persistent scapholunate
instability.

Scapholunate instability is the most common carpal instability. The primary


stabilizing structure of the scaphoid and lunate bones is the scapholunate ligament,
which is commonly injured with a fall on an outstretched hand. Secondary stabilizers
of the scaphoid and lunate include the dorsal intercarpal ligaments and the dorsal
radiocarpal ligaments. Failure to recognize injury of these structures can cause
persistent dorsal intercalated segment instability (DISI). This can predispose patients
to a SLAC wrist and early wrist osteoarthritis.

Mitsuyasu et al. examined the role of dorsal intercarpal ligaments (DIC) in


scapholunate instability. They showed that the DIC had an important role in
stabilizing the scaphoid and lunate bones with static and dynamic movements. The
authors of this study suggest that the DIC ligament should be assessed
intraoperatively and consideration should be given to repair and/or reconstruction
with surgical management of scapholunate ligament tears.

Viegas et al. showed that the dorsal intercarpal and the dorsal radiocarpal ligaments
form a lateral V configuration over the dorsal wrist. This configuration acts as an
indirect dorsal stabilizing effect on the scaphoid throughout the range of motion of the
wrist. Their integrity acts to ensure normal wrist kinematics.

Figure A shows an AP and lateral radiograph of the left hand. There is significant
gapping between the scaphoid and lunate articulation. This is indicative of a complete
scapholunate dissociation, however both wrists should be imaged as this deformity
may exist without injury. Illustration A shows the anatomy of the dorsal intercarpal
and the dorsal radiocarpal ligaments.

Incorrect Answers:
Answers 1, 3, 4, 5: Carpal ligamentous injuries can occur with falls onto an
outstretched hand. However, the clinical and radiographic features of this patient do
not show injury to these structures.

(SAE11OS.195) Which of the following proximal phalanx fractures can most reliably
be treated with a closed reduction and avoidance of surgical measures? Review Topic

1 Midshaft transverse diaphyseal fracture with 30 degrees of angulation

2 Long spiral diaphyseal fracture with 15 degrees of malrotation

3 Open fracture with skin loss and exposed extensor tendon

4 Distal condylar intra-articular fracture with minimal displacement

5 Proximal metaphyseal fracture location with 30 degrees of dorsal tilting


PREFERRED RESPONSE 1

Proximal phalanx fractures are very common, but care must be taken to understand
which injuries are reliably treated with nonsurgical measures, and which ones are
prone to clinically symptomatic malunion without surgical treatment. The proximal
metaphyseal location is a problematic fracture to get reduced with closed measures,
and due to the forces of the extensor apparatus, is prone to collapse into the original
deformity. Imaging is also frequently difficult because of the overlap of the other
fingers and frequently the true angulation is underappreciated. With 30 degrees of
angulation, consideration should be given to surgical treatment. Long oblique/spiral
fractures with malrotation are also most reliably treated with multiple lag screws,
because maintaining the reduction with nonsurgical measures is unreliable, and can
lead to significant functional problems in the form of crossover of the fingers with
gripping. Open fractures with skin loss clearly are treated with surgical measures.
Distal condylar fractures with minimal displacement are another fracture pattern that
have a high rate of loss of reduction when treated nonsurgically. Like most articular
fractures, they are best treated with anatomic reduction and rigid internal fixation. By
comparison, closed midshaft transverse diaphyseal fractures can usually be
anatomically reduced and held in this position with closed measures.

(SAE11AN.44) Which of the following constitutes a positive intrinsic tightness test?


Review Topic

1 Decreased proximal interphalangeal joint flexion with extension of the


metacarpophalangeal joint

2 Decreased proximal interphalangeal joint flexion with flexion of the metacarpophalangeal


joint

3 Normal proximal interphalangeal joint flexion with flexion of the metacarpophalangeal


joint

4 Normal proximal interphalangeal joint flexion with extension of the metacarpophalangeal


joint

5 Increased proximal interphalangeal joint flexion with extension of the


metacarpophalangeal joint
PREFERRED RESPONSE 1

Extension of the metacarpophalangeal joint places tension on the intrinsic


contribution to the extensor system via the lateral bands. Contracture of the intrinsics
decreases the flexion at the proximal interphalangeal joint with the
metacarpophalangeal joint placed in extension past neutral.

(SAE11OS.123) What is the most efficient pressure for use with negative pressure
wound therapy? Review Topic

1 25 mm Hg

2 75 mm Hg

3 125 mm Hg

4 300 mm Hg

5 500 mm Hg

PREFERRED RESPONSE 3

In animal and clinical studies, a range of pressures between 50 mm Hg to 500 mm Hg


were tested; the most efficient pressure was 125 mm Hg, resulting in a fourfold
increase in blood flow, 63% increase in granulation tissue with continuous pressure,
and 103% increase in granulation tissue with intermittent pressure. When 125 mm Hg
pressures were compared with either those less than 50, or those greater than 250,
there was a decrease in granulation tissue in swine models.

(SAE08OS.42) All of the following structures are components of the triangular


fibrocartilage complex. What structure is considered the most important restraint of
dorsal/palmar translation of the radius on the ulna? Review Topic

1 Triangular articular disk

2 Dorsal and palmar radioulnar ligaments

3 Ulnolunate ligament

4 Ulnotriquetral ligament

5 Subsheath of the sixth extensor compartment


PREFERRED RESPONSE 2

Clinical examination of the distal radioulnar joint is frequently described as


translation of the ulnar head in reference to the radius. This description is incorrect,
since the ulna is the bone that is fixed in space. Anatomically speaking, the radius
moves in relation to the fixed ulna. The triangular fibrocartilage complex is composed
of all of the structures listed above. The articular disk is a meniscal-like structure that
serves a load-bearing function between the ulnar carpal bones and the ulnar head.
About 20% of the load borne across the wrist passes through the disk. It has almost no
ligamentous (stabilizing) function. At the dorsal and volar margins of the articular
disk are thickened true ligamentous ligaments termed the radioulnar ligaments. They
take origin from the foveal area of the ulnar head and styloid and insert into the dorsal
and volar margins of the sigmoid notch. These two ligaments serve as primary
restraints to dorsal and palmar translation of the radius on the ulna. The ulnolunate
ligament, ulnotriquetral ligament, and the subsheath of the sixth extensor
compartment are ligaments, but they serve to stabilize the carpus to the ulna and
radius. They provide minimal stability to the distal radioulnar joint.

(SAE07SM.3) The most common mechanism of injury to the triangular fibrocartilage


complex (TFCC) involves Review Topic

1 wrist extension and forearm pronation.

2 wrist extension and forearm supination.

3 wrist flexion and forearm pronation.

4 wrist flexion and forearm supination.

5 axial load in ulnar deviation.

PREFERRED RESPONSE 1

TFCC tears are common in athletes. As the athlete braces for a fall, the wrist is most
commonly in an extended position and the forearm is pronated.
(OBQ13.65) A 30-year-old male laborer sustained a right wrist injury 9 months ago.
He continues to have symptoms of recurrent ulnar-sided wrist pain that impairs his
ability to work. An MRI is performed and shows a triangular fibrocartilage complex
(TFCC) injury. Which of the following is an indication to combine a Wafer procedure
with arthroscopic TFCC debridement? Review Topic

1 Ulnar styloid fracture

2 Radial styloid fracture

3 2 mm of positive ulnar variance and ulnocarpal impingment

4 2 mm of negative ulnar variance and radiocarpal joint arthritis

5 Scapholunate ligament injury

PREFERRED RESPONSE 3

A Wafer procedure is indicated for positive ulnar variance and symptomatic


ulnocarpal impingement associated with degenerative TFCC tears.

Ulnar impaction syndrome and triangular fibrocartilage complex (TFCC) injuries are
relatively common causes of ulnar-sided wrist pain. Positive ulnar variance causes
increased contact pressures between the lunate and the ulnar head. The Wafer
procedure removes 2-4 mm of distal ulnar head to reduce ulnar variance to neutral or
negative. This is thought to reduce ulnar impaction and decrease pain.

Faber et al. examined the role of MRI in wrist injuries. They showed that the
sensitivity and specificity to detect TFCC tears using MRI is approximately 80%.
They conclude that there is no supporting evidence for routine MRI's for patients with
non-specific ulnar-sided wrist pain.

Illustration A is a coronal view MRI (without arthrogram) of the right wrist that
shows a TFCC tear (blue arrow) with positive ulnar variance. Illustration B shows a
series of images showing a TFCC tear on MRI and intra-operatively.

Incorrect Answers:
Answers 1,2,4,5: Arthroscopic Wafer procedure is indicated for positive ulnar
variance and ulnocarpal impingement associated with TFCC tears. This procedure
uses the central TFCC tear to access the distal ulna with the arthroscope. The
pathological findings supplied in the other answer options are not indications for this
surgical procedure.
(SAE11UE.37) A patient has severe cubital tunnel syndrome and marked wasting of
the intrinsic muscles of the hand. Why is the little finger held in an abducted position?
Review Topic

1 Accessory slip of the extensor digiti minimi attaching to the abductor digiti minimi tendon

2 Tetanic contraction of the abductor digiti minimi

3 Radial collateral ligament insufficiency of the fifth metacarpophalangeal (MCP) joint

4 Unopposed pull of the flexor digitorum profundus

5 Muscle innervation from a Martin-Gruber anastomosis

PREFERRED RESPONSE 1

A Wartenberg's sign, where the little finger is held in an abducted position, is


associated with an ulnar nerve palsy. This happens when there is an accessory slip of
the extensor digiti minimi, which is innervated by the radial nerve, crossing ulnar to
the center of the MCP joint to attach to the tendon of the abductor digiti minimi and
the proximal phalanx. The abductor digiti minimi and the volar interosseous muscles
are both innervated by the ulnar nerve; therefore, there is no tetanic contraction of the
abductor digiti minimi. Unopposed pull of the flexor digitorum profundus results in
excess flexion of the proximal interphalangeal and distal interphalangeal joints of the
hand as seen with a clawing-type deformity. A Martin-Gruber anastomosis, which is a
neural connection between the ulnar and median nerves in the forearm, cannot explain
this finger position.

(SAE08UE.115) Which of the following is considered an important component in


treating the lesion shown in Figure 56? Review Topic

1 Excision of the skin in addition to the cyst


2 Resection of the nail plate

3 Excision of bony osteophytes from the distal interphalangeal (DIP) joint

4 Injection of corticosteroid into the DIP joint

5 Resection of part of the collateral ligament and extensor mechanism


PREFERRED RESPONSE 3

Mucoid cysts are commonly associated with DIP joint arthritis. Two treatment options
are commonly used: (1) aspiration/drainage and injection of corticosteroid and (2)
surgical excision. When performing the surgery, excision of the bony osteophytes
about the DIP joint is helpful in achieving a cure. There are no reports of significant
benefit with nail removal or partial ligament or extensor tendon resection. Some
authors have advocated skin excision and rotational flaps for wound coverage, but this
is somewhat controversial.

(OBQ15.274) Demyelination of a nerve fiber will result in which of the following on


a nerve conduction velocity study (NCS)? Review Topic

1 Increased amplitude

2 Increased latency

3 Normal evoked reponse

4 No change in the conduction velocity

5 No change in latency

PREFERRED RESPONSE 2

Demyelination of a nerve fiber results in increased latency on a nerve conduction


velocity test.

A nerve conduction velocity test is a measurement of the speed of conduction of an


electrical impulse through a large, myelinated nerve. In this test, evoked responses are
measured after stimulation of a peripheral nerve. Typical measurements include the
nerve conduction velocity, amplitude, and duration. When a nerve becomes
demyelinated, or damaged, there is increased latency and overall decrease in the
conduction velocity. By analyzing the results, one can differentiate between a normal
nerve, demyelination, axonotmesis, neurotmesis, and neuropraxia. Common
indications include carpal and cubital tunnel, radiculopathy, and nerve dysfunction of
the shoulder. A thicker myelin sheath will affect nerve transmission by increasing the
speed of wave propagation.
Wilbourne reports that the electrodiagnostic exam (including nerve conduction
studies) detect demyelination injuries along large, myelinated motor axons, as well as
the sensory axons for position, vibration, and some light touch. Demyelination of the
axon results in a block to conduction, focal slowing of signal transmission, and
manifests as increased latencies on NCS stimulation proximal to the lesion. Clinically,
focal demyelination manifests as decreased deep tendon reflexes, loss of vibrational
sense, and intermittent pain and paresthesias. It does not, however, affect motor
strength or sensibility.

Illustration A shows a diagrammatic representation of a nerve conduction velocity


test.

Incorrect Answers:
Answer 1: Demyelination results in decreased amplitude.
Answer 3: Demyelination results in an absent or prolonged evoked response.
Answer 4: Demyelination results in a decrease in conduction velocity.
Answer 5: Demyelination results in increased latency.

(SAE11OS.77) A 32-year-old woman jammed her ring finger. Figures 77a and 77b
show radiographs of the finger after a closed reduction. Which of the following
interventions, if done correctly, is likely to result in the best possible final clinical
outcome? Review Topic

1 Early removal of a splint and application of continuous passive motion


2 Application of dynamic extension bracing after the first week

3 Maintaining reduction of the middle phalanx on the condyles of the proximal phalanx with
dynamic external fixation

4 Open reduction and anatomic restoration of the middle phalanx articular surface

5 Surgical advancement of the volar plate into the middle phalanx base
PREFERRED RESPONSE 3

The most important determinant in the final clinical outcome in proximal


interphalangeal (PIP) joint fracture locations is the maintenance of the PIP joint
alignment on the lateral view. This can sometimes be done with just extension block
splinting, sometimes the fracture requires dynamic external fixation, and sometimes
the fracture requires open reduction or volar plate arthroplasty. Good function can be
the result in the setting of an incongruent middle phalanx base as long as the PIP joint
alignment is maintained. Continuous passive motion has not been shown to be of
benefit. Whereas dynamic external fixation in a flexed position is a very good
treatment, dynamic extension bracing will just precipitate loss of PIP joint reduction
and is therefore not indicated. Whereas open reduction of the articular surface is
theoretically desirable, it is generally impossible in the setting of the comminution of
the volar middle phalanx base. Furthermore, open reduction and internal fixation by
itself does not guarantee that the PIP joint alignment will be maintained, and typically
it causes finger stiffness given the extensive surgical approach. Likewise, volar plate
arthroplasty is a surgery of last resort and requires careful attention to PIP joint
alignment before joint pinning. In this case, with characteristics of comminution,
dynamic external fixation is the preferred choice.

(OBQ05.224) A 8-month-old male presents with the hand deformity seen in Figure A.
Examination reveals restrictions in passive extension and abduction of the thumb. His
mother tells you that her other two children had similar deformities that required
operative management. What would you offer as the first-line treatment option for
this child's deformity? Review Topic
1 Observation
2 Serial splinting of thumb

3 Thumb-index finger web space reconstruction

4 Thenar muscle tendon transfer

5 First metacarpophalangeal joint arthrodesis

PREFERRED RESPONSE 2

This patient is presenting with a congenital clasped thumb. The first line treatment for
all types of congenital clasped thumb is serial splinting of thumb for 3-6 months.

Congenital clasped thumbs are flexion-adduction deformities of the thumb that persist
beyond the 3rd or 4th month of life. They usually result from the absence or
hypoplasia of the extensor pollicis brevis (EPB) and/or the extensor pollicis longus
(EPL). The deformity was categorized by Tsuyugushi from Type I - III based on
anatomy of deformity. All types are initially managed conservatively with serial
splinting for 3-6 months. Surgical treatment with first webspace widening and tendon
transfer are considered after failed conservative management.

Tsuyuguchi et al. reviewed 75 hands with congenital clasped thumb. Three groups
were identified based on presenting deformities. All patients with type I deformities
showed good results with splinting. Type II and III deformities had poor results to
splinting and required operative management.

Hisham et al reviewed the characteristics in 40 patients with congenital clasped


thumb. They concluded that conservative management should be initiated in all Type
I and II patients initially.

Ruland and Slakey do an excellent job of reviewing the critical differences between
trigger thumb and congenital clasped thumb.

Figure A shows a moderate clasped thumb deformity (Type II deformity).

Incorrect Answers:
Answer 1: The deformity is unlikely to correct with observation
Answer 3: Thumb-index finger web space reconstruction is considered for Type II or
III deformities when there is significant contracture of the skin in the web space.
Answer 4: Thenar muscle tendon transfer can be considered with failed treatment of
type I or II deformities.
Answer 5: First metacarpophalangeal joint arthrodesis is a last resort treatment for
deformities associated with significant MP joint instability and deformity.

(SAE07SM.78) A 40-year-old right-handed professional football player reports


persistent right wrist pain after falling during a game 5 days ago. A radiograph is
shown in Figure 21. Management should consist of Review Topic

1 immobilization in a short arm thumb spica cast.


2 immobilization in a long arm thumb spica cast.

3 arthroscopic repair and percutaneous pinning.

4 open repair and percutaneous pinning.

5 dorsal capsulodesis.

PREFERRED RESPONSE 4

The radiograph reveals an increased distance between the scaphoid and the lunate,
which is indicative of scapholunate disassociation. A ring sign is also present, which
represents the distal pole of the scaphoid viewed end on in a palmarly flexed position.
In the acute setting, the scapholunate can be repaired. Open repair and percutaneous
pinning is the treatment of choice. Dorsal capsulodesis is performed in the chronic
setting if such an injury is initially missed.
(OBQ14.65) A hand surgeon is about to repair a zone II flexor tendon rupture in the
middle digit. He decides to perform wide-awake repair. Which of the following is true
about wide-awake tendon repair? Review Topic

1 It is safe to perform an injection of local lidocaine with epinephrine into the hand and digit

2 Surgery is performed under brachial plexus block with sedation

3 A forearm tourniquet is used instead to prevent arm tourniquet discomfort

4 Gaps are revealed by the surgeon passively flexing the digit through the total arc of motion

5 Wide-awake tendon repair has increased the rate of identification of gaps, but has not
reduced the need for late post-operative tenolysis.

PREFERRED RESPONSE 1

Wide-awake flexor tendon repair uses local lidocaine and epinephrine injections.

Epinephrine is now known to be safe when used in the digits and hand and does not
cause necrosis even at high doses (up to 1:1000). Finger necrosis previously blamed
on epinephrine is known to be caused by procaine. The antidote phentolamine reliably
reverses epinephrine vasoconstriction.

Lalonde et al. recommend tumescent local anesthesia (large volume, low


concentration). For < 50 mL injections, they use 1% lidocaine with 1:100,000
epinephrine. If 50-100 mL is needed, they use 0.5% lidocaine with 1:200,000
epinephrine. If 100-200 mL is needed (large forearms), they use 0.25% lidocaine with
1:400,000 epinephrine. They perform the repair with 3-0/4-0 Ethibond Kessler sutures
(4-strand) and running 6-0 nylon epitendinous sutures.

Lalonde describe postoperative therapy after wide-awake tendon repairs. They


immobilize and splint the hand for 3 days. After the 3rd day, they allow midrange
active motion, allowing patients to make a half a fist: 45 degrees of active flexion at
MP, PIP, and DIP joints, but not full flexion of PIP and DIP joints because friction
across the repair and the risk the repair catching on a pulley edge might lead to
rupture.

Illustration A shows the sequence of injections (left to right) of lidocaine-epinephrine


for wide-awake tendon surgery.

Incorrect Answers:
Answer 2: Wide-awake surgery is done only under local anesthesia without sedation.
Answer 3: No tourniquet is used. Bleeding is reduced by using the admixture of
epinephrine and lidocaine.
Answer 4: Gaps are revealed by the awake patient ACTIVELY flexing and extending
through the full range of motion intraoperatively. This allows on-the-spot
identification of gaps and tightening of the repair. Without a tourniquet, the surgeon is
able to performed a repair without rush as there is no tourniquet pain.
Answer 5: Wide-awake repairs allow identification of repairs that will not fit through
the pulleys because if they do not fit through pulleys during surgery, they will not fit
through after surgery. This allows for debulking of bulky repairs, division of the A4
and venting (partial division) of the A2 pulleys. These actions have reduced the need
for late tenolysis.

(SAE11AN.87) Contracture or tightness of the triangular ligament of the finger is


associated with which of the following conditions? Review Topic

1 Sagittal band insufficiency

2 Volar subluxation of the lateral bands

3 Swan-neck deformity

4 Volar plate contracture

5 Boutonniere deformity

PREFERRED RESPONSE 3

Swan-neck deformity may result from contracture/tightness of the triangular ligament.


Anatomically, the triangular ligament is on the dorsal aspect at the base of the middle
phalanx just distal to the central slip. It keeps the lateral bands dorsal. With a
boutonniere deformity, the lateral bands move volar to the central axis resulting in a
flexion deformity of the proximal interphalangeal joint and extension of the distal
interphalangeal joint. Sagittal band insufficiency results in subluxation of the extensor
tendon(s) at the metacarpophalangeal joint level. Volar plate contracture will not
cause swan-neck deformity.
(SAE09FA.75) Which of the following conditions precludes performing a tendon
transfer? Review Topic

1 The target joint has a full passive range of motion.

2 The range of motion of the target joint only occurs in the direction of correction.

3 The target joint cannot be passively corrected to its neutral position.

4 The muscle to be transferred is out-of-phase.

5 There is no pulley to assist the transferred muscle’s fulcrum.

PREFERRED RESPONSE 3

Several conditions must be met before a tendon transfer has the potential to correct a
dynamic deformity. If the target joint cannot be passively corrected to neutral, it
indicates that a static joint contracture or bony deformity exists that cannot be
corrected with a dynamic tendon transfer. While in-phase muscles are best, out-of-
phase muscles are often the only muscles available for transfer. Tendon transfer
should pull in a straight line to avoid tethering and late failure.

(OBQ14.228) A patient with Dupuytren's disease enquires about percutaneous needle


fasciotomy and collagenase injections. Which of the following is true regarding these
two treatment modalities? Review Topic

1 Collagenase injections are more expensive but require fewer treatments than
percutaneous needle fasciotomy to achieve the same results

2 Softening of Dupuytren cords and nodules is observed in cadaveric experiments after


collagenase injections but not in a clinical setting

3 Skin tears are seen following percutaneous needle fasciotomy but not after collagenase
injections

4 Pruritic rash and axillary lymphadenopathy may be observed after collagenase injections
but are not typically seen after percutaneous needle fasciotomy

5 Percutaneous needle fasciotomy has a higher incidence of iatrogenic flexor tendon rupture
PREFERRED RESPONSE 4

Pruritic rash and axillary lymphadenopathy is particular only to collagenase injections


(CI). These reactions do not occur after percutaneous needle fasciotomy (PNF).

Regional subtotal fasciectomy is the gold standard of care for treatment of


Duyputren's contracture. However, because of high complication rates (up to 40%),
nonoperative treatments are attractive. In PNF, a 22G or 25G needle is used to release
palpable/visible cords in the office. This is followed by manipulation, and night
orthosis wear. In CI, 0.25ml of collagenase is used for MCP contracture and 0.20ml
for PIP contracture. Manipulation is performed the following day under local
anesthesia. This is repeated at 4 weeks if the desired result is not achieved.

Black et al. reviewed Dupuytren's disease. They note that recurrence rates were higher
for PNF (85%) compared with fasciectomy (24%) and conclude that PNF is
acceptable for early disease with less severe contracture and elderly, sick patients.
Regarding CI, they note that the most common complications are edema, contusion,
pain, lymphadenopathy, and skin laceration. Major complications, such as complex
regional pain syndrome and flexor tendon rupture, are rare.

Nydick et al. retrospectively compared PNF (30 patients) with CI (29 patients). Both
groups achieved similar satisfaction, complication rates, and success rates defined as
contracture reduction to 0-5 degrees. They concluded that both PNF and CI provide
good alternatives to surgery.

Incorrect Answers:
Answer 1: Collagenase injections are more expensive and more injections are needed
to achieve the same results as PNF.
Answer 2: Softening of Dupuytren cords and nodules is seen in the clinical setting and
is one of the reported short term advantages of CI.
Answer 3: Skin tears are a result of forceful manipulation following either CI or PNF,
and not a result of the injection or fasciotomy per se. Skin tears are therefore a
complication of both treatments.
Answer 5: No major complications such as tendon rupture, nerve, or vessel injury
were observed in either group in the Nydick et al study.

(SAE11UE.44) An 11-year-old boy sustained a fall onto his outstretched right hand
while playing soccer. Examination reveals tenderness in the anatomic snuff box.
Wrist radiographs reveal a scaphoid fracture. This injury most commonly presents
with which of the following? Review Topic

1 Within the distal one third of the scaphoid

2 Within the middle one third of the scaphoid

3 Within the proximal one third of the scaphoid

4 In association with injury to the scapholunate ligament

5 As a unicortical injury
PREFERRED RESPONSE 1

The distal pole of the scaphoid ossifies before the proximal pole, resulting in an
increased incidence of distal one third fractures and avulsions of the distal radial
aspect of the scaphoid (59% to 94%) as compared with adults. Scaphoid fractures in
the pediatric population can be seen in association with distal radius fractures, but are
not commonly associated with ligamentous injury. While 23% of pediatric scaphoid
fractures are unicortical, bicortical injuries still predominate. Most pediatric scaphoid
fractures are nondisplaced and heal with 4 to 6 weeks of immobilization. As in adults,
displaced fractures are treated with open reduction and internal fixation.

(OBQ14.175) A 27-year-old motorcyclist is thrown from his vehicle and sustains an


open radiocarpal dislocation. The injury is reduced in the emergency room and a post-
reduction radiograph is shown in Figure A. The main volar extrinsic ligament that
must be disrupted to cause this appearance is the Review Topic

1 Short radiolunate ligament


2 Scapholunate interosseous ligament

3 Radiotriquetral ligament

4 Radioscaphocapitate ligament

5 Radioscapholunate ligament

PREFERRED RESPONSE 4

The stout radioscaphocapitate ligament is the main restraint against ulnar translation
of the carpus.
On a posteroanterior radiograph, carpal alignment is evaluated using the lunate
position relative to the radius. The lunate should align with the ulnar column of the
distal radius with a minimum of 2/3 of the lunate articulating with the radius.
Radiocarpal fracture dislocations can be classified based on the presence of associated
intercarpal dissociation (Moneim classification), or the extent of radial styloid
involvement (Dumontier classification).

Ilyas et al. reviewed radiocarpal fracture dislocations. They advise suspecting a


scapholunate injury when the radial styloid fracture exits at the junction between
scaphoid and lunate fossae. They enumerate the steps for surgical treatment as: (1)
provisional reduction, (2) neurovascular decompression, (3) joint de´bridement, (4)
treatment of intercarpal injuries, and (5) fracture fixation and/or soft tissue repair.

Mudgal et al. retrospectively reviewed 12 patients with radiocarpal fracture


dislocations. Open injuries and Moneim Type II injuries result in inferior outcomes
compared with closed and Type I injuries. They opine that anatomical repair and
stable fixation will lead to a satisfactory outcome in these injuries.

Figure A shows ulnar translation of the carpus. The lunate is now centered over the
distal ulna instead of over the lunate fossa of the radius. Illustrations A and B are the
Moneim and Dumontier classifications of radiocarpal fracture-dislocations
respectively. Illustration C shows the volar extrinsic ligaments. The main ligaments
seen are
the radioscaphocapitate (RSC), the radioscapholunate (RSL), long radiolunate (or
radiolunotriquetral or volar radiolunate), and short radiolunate.

Incorrect Answers:
Answer 1: The short radiolunate ligament is likely disrupted. However, while it
functions to add mechanical support to the lunate (and in turn, the rest of the carpus),
it is not the main mechanical restraint to ulnar translation of the carpus.
Answer 2: The SLIL is an intrinsic ligament. Disruption leads to a widened SL
interval and "Terry Thomas sign" on PA radiograph.
Answer 3: The radiotriquetral ligament is a dorsal extrinsic ligament. Disruption of
this structure (together with lunotriquetral ligament rupture) leads to VISI deformity.
Answer 5: The radioscapholunate ligament (Ligament of Testut and Kuentz) is a
neurovascular conduit and confers no mechanical strength.

(SAE08OS.79) A 38-year-old man who works as an auto mechanic reports right wrist
pain and intermittent paresthesias and numbness into the ulnar two digits of his hand.
The pain symptoms begin after a day of particularly strenuous repetitive use of the
right hand. He denies any color changes to the digits but does report a significant
smoking history. Weakness of the interossei muscles are seen on manual testing. Both
radial and ulnar pulses are palpable at the wrist. The patient states that sensation on
the dorsum of his ring and little fingers is normal. Radiographs are shown in Figures
25a through 25c. What is the most likely diagnosis? Review Topic

1 Idiopathic compression of the ulnar nerve at Guyon's canal


2 Ulnar hammer syndrome

3 Cubital tunnel syndrome

4 Pseudoaneurysm of the ulnar artery in the hand

5 Nonunion of the hamate

PREFERRED RESPONSE 5

The anatomy of the distal ulnar tunnel is divided into three zones. Zone 1 is proximal
to the bifurcation of the ulnar nerve and consists of both sensory and motor fibers of
the nerve. Zone 2 represents the motor branch of the ulnar nerve distal to the
bifurcation. Zone 3 represents the sensory branches of the ulnar nerve beyond its
bifurcation. The site or zone of compression will predict the clinical presentation. The
most common causes of compression in zones 1 and 2 are ganglions and fractures of
the hook of the hamate, which can be seen on the radiographs. The patient has
significant risk factors for ulnar vascular disease as a root cause of his symptoms. The
most common cause of isolated sensory nerve branch compression in zone 3 is ulnar
artery thrombosis, but this does not fit with this patient because he has both motor and
sensory findings. The dorsum of the fingers have normal sensation; therefore, the
cubital tunnel is unlikely as a cause of his symptoms. Idiopathic compression is ruled
out by the nonunion of the hamate which is clearly seen on the oblique view, just
distal to the pisiform.

(SAE08UE.90) What is the most appropriate surgical treatment for a stage III
symptomatic scapholunate advanced collapsed (SLAC) wrist? Review Topic

1 Radioscapholunate arthrodesis

2 Scaphotrapeziotrapezoid arthrodesis

3 Scaphocapitate arthrodesis

4 Proximal row carpectomy

5 Scaphoid excision and capitate-lunate-triquetrum-hamate arthrodesis

PREFERRED RESPONSE 5

SLAC is the end result of chronic scapholunate instability. The arthritis follows a
predictable pattern. Stage I disease involves cartilage loss between the waist of the
scaphoid and the radial styloid. In stage II, the arthritis progresses to include the
proximal pole of the scaphoid and the scaphoid fossa of the radius. Finally, stage III
goes on to include arthritis of the capitolunate joint. The only treatment option that
addresses all of the sites of arthritis is the scaphoid excision and four corner fusion.

(OBQ14.62) Figures A and B are the radiographs of a 15-year-old boy with a


congenital condition of the left elbow. His forearm is fixed in 75 degrees of pronation.
If operative treatment is undertaken with the goal of restoring motion, what step
should be included in the procedure to prevent recurrence of the condition? Review
Topic
1 Soft tissue reconstruction
2 Interposition of material between radius and ulna

3 Derotational osteotomy to fix the forearm in neutral position

4 Postoperative radiation

5 Postoperative casting

PREFERRED RESPONSE 2

This patient has a congenital radioulnar synostosis. Recurrence of synostosis is


common without interposition of something between the radius and ulna

Radioulnar synostosis is a rare congenital anomoly may cause complete loss of


forearm rotation with anklyosis in a position anywhere from neutral to
hyperpronation. Physical exam is characteristic and xray confirms the diagnosis. Most
cases are treated non-operatively. If excision of the synostosis is undertaken,
something must be done to reduce the risk of recurrence, such as interposition of a
tissue that does not form bone or scar (e.g. free fat, pedicled fat, muscle, Gelfoam,
Micropore). The other surgical option is derotational osteotomy, which does not
restore motion but may be used to fix the forearm in a more functional position.

Cleary et al. evaluated 23 patients with forearms fixed in an average of 30 degrees of


pronation. They found that most patients had few or no functional limitations and
were employed in jobs that demanded heavy forearm use. They concluded that
operative treatment is rarely indicated and that functional tests rather than forearm
position should be emphasized.

Kanaya et al. excised the synostosis of 7 patients (average age 8 years) using fat graft
interposition. Average post op supination was 26 degrees. There were no cases of
recurrent ankylosis. Their article includes a nice write up of the operative technique.
Their review of the literature found that mobilization procedures without interposition
failed (recurrent synostosis) in all but one of 22 patients (23 forearms).
Figures A and B show anterior-posterior (AP) and lateral views of the elbow
demonstrating proximal radio-ulnar synostosis. Illustrations A and B show AP and
lateral views of the elbow demonstrating proximal radio-ulnar synostosis and
posterior radial head dislocation. Illustrations C and D show postoperative
radiography of the same patient.

Incorrect answers:
Answer 1: Reconstruction of the capsule, annular ligament or biceps tendon may be
indicated depending on the deformity but will not affect recurrence of the synostosis.
Answer 3: Although this is an accepted treatment for this procedure, the stem asks
about treatment to restore motion.
Answer 4: This is not described for this condition.
Answer 5: Immobilization may be utilized in postoperative period but will not prevent
recurrent synostosis.

(OBQ15.28) A 32-year-old male sustains a 75% laceration of the flexor digitorum


profundus tendon of the index finger of his dominant hand and presents with
triggering. Which of the following techniques uses the least amount of suture
necessary to prevent gap formation >2mm? Review Topic

1 No repair

2 Core suture repair

3 Epitendinous suture repair of the cut edges only

4 Circumferential epitendinous suture repair

5 Core and epitendinous suture repair

PREFERRED RESPONSE 3

Epitendinous suture repair of the cut edges is all that is necessary to prevent gap
formation.

Most surgeons repair lacerations of = 50% if there is triggering and nonrepair for 50-
60% lacerations without triggering. Although lacerations of up to 75% are capable of
handling early, non-resisted active mobilization, most hand surgeons will repair 75%
partial lacerations.

Haddad et al. compared repair vs nonrepair of 75% lacerated sheep flexor tendons.
They found that peripheral or peripheral + core repairs reduced gap formation (= 1mm
at 500 cycles) but there was no difference in gap formation between these 2 groups.
Nonrepaired tendons had gap formation early (100 cycles). They concluded that
gapping is reduced with a peripheral repair (with or without core suture).

Illustration A shows epitendinous sutures at the laceration site.

Incorrect Answers:
Answer 1: Gap formation occurs rapidly with no repair.
Answer 2: Core suture repairs do not provide adequate apposition at the cut edge to
prevent gapping.
Answer 4: For partial lacerations, there is no advantage to circumferential
epitendinous sutures compared with epitendinous sutures at the lacerated edge alone.
Answer 5: The addition of core sutures increases bulk of the repair and is unnecessary
to prevent gapping. In the study above by Haddad et al., there was no difference
between core+epitendinous and epitendinous alone for partial lacerations.

(OBQ14.271) A 50-year-old woman complains of severe pain and clicking in her


right thumb after opening a jar 1 year ago. She recalls a history of steroid injection for
a chronic tenosynovitis of the flexor pollicis longus tendon. Examination reveals
diminished range of thumb motion. A clinical photograph is shown in Figure A. An
MRI scan is shown in Figure B. An intraoperative photograph is shown in Figure C.
What is the diagnosis? Review Topic
1 Pulley rupture
2 Radial collateral ligament rupture

3 Flexor pollicis longus musculotendinous avulsion

4 Flexor pollicis longus bony avulsion

5 Gout

PREFERRED RESPONSE 1

This patient has a rupture of the oblique pulley.

The fingers have 5 annular and 3 cruciate pulleys. Closed flexor pulley rupture in the
fingers is uncommon and happens in rock climbers. The thumb has 2 annular (A1 and
A2) and 1 oblique pulley, and 1 variable annular pulley (Av). The arrangement is A1-
Av-oblique-A2. Pulley ruptures in the thumb are rare. Loss of both oblique and
proximal pulleys leads to FPL bowstringing. Treatment involves pulley reconstruction
with a tendon graft. Reconstructing the oblique pulley alone is enough to prevent
bowstringing.

Bayat et al. explored the pulleys of the thumb using cadavers. The A1 and Av pulleys
are 6mm long. The oblique pulley originated from the ulnar side of the proximal
phalanx base and inserted to the radial side of the distal phalanx base and was 4mm
wide. The oblique pulley experiences greater strain in extension than in flexion. The
A2 pulley was 8mm wide. When both A1 and Av pulleys have been sectioned, the
oblique pulley can no longer prevent bowstringing.

Kosiyaktrakul et al described a case report of A1, oblique, and A2 pulley rupture,


leading to bowstringing. This was successfully treated with pulley reconstruction of
the oblique pulley alone.

Figure A shows bowstringing of the FPL tendon. Figure B is an MRI image showing
volar subluxation of the FPL tendon off the proximal phalanx (they should be closely
apposed). Figure C is an intraoperative photograph showing separation of the FPL
tendon (volar) from bone (dorsal). Illustration A shows the 3 different types of Av
pulley (left, Type I, with gap between A1 and Av; center, Type II, with no gap; right,
Type III, oblique orientation).

Incorrect Answers:
Answer 2: There is no rupture of the radial collateral ligament.
Answers 3 and 4: The FPL is grossly intact. Bowstringing (tension) can only occur in
the presence of an intact tendon that is fixed at both ends (proximal and distal).
Answer 5: There are no deposits to suggest gout. Gouty attrition can occasionally lead
to tenosynovitis and tendon rupture. This is not seen in this case.
(SAE08AN.45) New painful paresthesias near the site of the incision after an ulnar
nerve transposition is the result of injury to what nerve? Review Topic

1 Medial antebrachial cutaneous

2 Lateral antebrachial cutaneous

3 Posterior antebrachial cutaneous

4 Medial brachial cutaneous

5 Dorsal antebrachial cutaneous

PREFERRED RESPONSE 1

Branches of the medial antebrachial cutaneous nerve can often be identified during
routine ulnar nerve surgery crossing the medial aspect of the elbow. It should be
preserved to avoid development of painful paresthesias.

(SAE11UE.101) Which of the following structures cannot be seen during standard


radiocarpal arthroscopy? Review Topic

1 Scapholunate ligament

2 Lunotriquetral ligament

3 Radioscaphocapitate ligament

4 Extensor carpi ulnaris tendon

5 Superficial insertion of the triangular fibrocartilage complex (TFCC)

PREFERRED RESPONSE 4

The extensor carpi ulnaris tendon is located in an extra-articular position, and as such,
cannot be seen during arthroscopy. Wrist arthroscopy is a useful technique for
evaluation and treatment of radiocarpal and midcarpal maladies. During standard
radiocarpal arthroscopy, the scapholunate and lunotriquetral ligaments can be easily
visualized. The superficial TFCC is seen overlying the ulnar head. Volarly, the
radioscaphocapitate ligament can be seen as a discrete band of the capsule.

(OBQ15.271) A 50-year-old man presents with basal thumb pain and weakness. He is
found to have Stage IV osteoarthritis (OA) of the carpometacarpal joint (CMC) of the
thumb. He undergoes Weilby suspension arthroplasty using the flexor carpi radialis
tendon (FCR). He returns 1 year later with recurrent pain. Radiographs are shown in
Figure A. What is the next best step? Review Topic

1 Trapeziometacarpal arthrodesis
2 Weilby suspension arthroplasty with extensor carpi radialis longus (ECRL) tendon

3 Ligamentous reconstruction tendon interposition (LRTI) with FCR tendon

4 Prosthetic arthroplasty

5 LRTI with ECRL tendon

PREFERRED RESPONSE 5

This patient has progressive subsidence of the thumb metacarpal after Weilby
suspensionplasty. LRTI with ECRL is the best salvage option.

There are many surgical options for CMC OA. This includes volar ligament
reconstruction and closing wedge dorsal extension osteotomy for early Stage I
disease. Late disease is treated with trapeziectomy with/without metacarpal base
stabilization, CMC fusion and prosthetic arthroplasty. There is no difference in results
of trapeziectomy alone vs trapeziectomy with LRTI. Silastic arthroplasty is
complicated by silicone synovitis. Pyrocarbon prosthetics are complicated by
subluxation attributed to a shallow trapezial cup. Fusion is indicated for young (<50y)
high demand patients who wish to maintain grip strength.

Jones et al. reviewed salvage options for FCR disruption during LRTI. For partial
FCR injury, they recommend using the remaining tendon for a Weilby
suspensionplasty, or the entire FCR for LRTI (if enough length is available). For
complete FCR avulsion, they recommend ECRL LRTI.

Conolly reviewed 17 patients undergoing revision for CMC OA (12 silastic


arthroplasty, 4 trapeziectomy, 1 arthrodesis). Revision options comprised implant
removal or replacement, soft tissue arthroplasty and fusion. 53% had good results,
30% had poor results. They conclude that trapeziectomy relieves pain but late
proximal migration and metacarposcaphoid arthritis occurs. Soft tissue interposition
may prevent this.

Figure A shows subsidence of the thumb metacarpal 1 year after trapeziectomy and
Weilby suspensionplasty. In CMC arthritis, the MP joint compensates with
hyperextension to allow for thumb pinch and thus leads to a zigzag ("Z") deformity.
When present, temporary pinning or fusion can be performed to stabilize the joint.
Illustration A shows LRTI with ECRL augmented with a suture button. Illustrations B
and C show the creation of an ECRL LRTI. Illustration D shows creation of an FCR
LRTI. Illustration E shows a Weilby suspensionplasty (with FCR). The Weilby
suspensionplasty involves a partial FCR slip woven through the abductor pollicis
longus (APL) tendon to stabilize the joint and suspend the thumb metacarpal without
drill holes.

Incorrect Answers:
Answer 1: The trapezium has been resected (trapeziectomy). This is not an option.
Answer 2: An ECRL based Weilby suspension is not the 1st choice salvage option.
Answer 3: The FCR tendon has already been used in the Weilby procedure and will
not be available for LRTI.
Answer 4: Prosthetic arthroplasty is not a good salvage option.

(SAE11UE.8) A 55-year-old woman with rheumatoid arthritis reports that she awoke
with an inability to flex the interphalangeal joint of her thumb. Figure 8 shows an
intraoperative finding. What is the most appropriate surgical treatment? Review Topic
1 Primary repair of the tendon
2 Tendon reconstruction with the palmaris longus tendon

3 Tendon reconstruction using a transfer of the flexor digitorum profundus (FDP) of the ring
finger

4 Thumb metacarpophalangeal fusion

5 End-to-side repair of the flexor pollicis longus to the FDP of the index finger

PREFERRED RESPONSE 2

The patient has sustained a chronic flexor pollicis longus rupture (Mannerfelt lesion).
The injury is most likely a result of tendinopathy and attritional rupture of the tendon
secondary to synovitis and bony osteophytosis at the scaphotrapeziotrapezoid joint.
Because of the attritional injury and inherent tendinopathy, primary repair is unlikely
to be successful. Among the options listed, tendon graft reconstruction with the
palmaris longus tendon is the most appropriate treatment. Tendon reconstruction is
possible with the flexor digitorum profundus of the index finger, not the flexor
digitorum profundus of the ring finger. If osteophytes are encountered, these should
be debrided. Thumb interphalangeal fusion is an option, but metacarpophalangeal
fusion is not beneficial. End-to-side repair of the flexor pollicis longus to the FDP of
the index finger is not appropriate and would sacrifice needed function of the index
finger.

(OBQ15.140) Which of the following carpal pathologies would be best treated with
medial femoral condyle vascularized bone grafting and open fixation? Review Topic

1 Chronic lunate osteochondral defect

2 Acute scaphoid fracture with humpback deformity

3 Proximal scaphoid avascular necrosis

4 Scaphoid nonunion advanced collapse with scaphocapitate arthrosis

5 Stage I Kienböck’s disease


PREFERRED RESPONSE 3

Out of the available options, vascularized medial femoral condyle graft and fixation is
most commonly used in the treatment of proximal scaphoid avascular necrosis and
scaphoid nonunion.

Free vascularized bone grafts (VBGs) are often considered in the management of
carpal bone nonunion. Various donor sites exist including the distal radius, iliac crest,
ribs and femoral condyles. The benefits of VBG techniques include increased bone
perfusion over time, accelerated graft consolidation, and rapid repopulation by cells.
However, a relative contraindication for their use is in the setting of carpal bones
without an intact cartilaginous shell, or advanced carpal collapse with degenerative
changes.

Al-Jabri et al. reviewed the use of the free vascularised bone graft for nonunion of the
scaphoid. They noted that the union rate in medial femoral condyle VBGs to be 100%
in 56 patients. They state the descending genicular vessels are most favoured due to
their longer nature and their wider caliber.

Jones et al. reported on the treatment of scaphoid waist nonunions with associated
avascular proximal pole and carpal collapse, with a medial femoral condyle bone
grafting. They showed better clinical results with medial femoral condyle bone
grafting compared with distal radius VBGs.

Illustration A shows vascularized medial femoral condyle graft harvesting using the
superomedial geniculate artery pedicle.

Incorrect Answers:
Answer 1: Chronic lunate osteochondral defect may be treated non-operatively with
observation only or arthroscopic osteochondral grafting with graft harvested from the
lateral femoral condyle.
Answer 2: Acute scaphoid fractures with humpback deformity rarely require free
vascularized bone grafts with initial treatment.
Answer 4: Scaphoid nonunion advanced collapse with scaphocapitate arthrosis is a
relative contraindication for free vascularized bone grafts and wrist salvage
procedures. Pan carpal arthritis is usually managed with arthrodesis.
Answer 5: Stage I Kienböck’s disease does not require open fixation techniques.
VBGs can be utilized for stage II disease and potentially for stage III(a) disease. The
most commonly used VBG for this disease is based on the fourth and fifth extensor
compartmental arteries (4 + 5 ECA), although many other techniques have been
described.
(OBQ13.75) A 27-year-old male sustains the injury shown in Figure A. He is taken to
the operating room and the lesion is repaired primarily. Two months later, he feels a
"pop" while using his hand and is no longer able to flex the distal phalanx of the
involved digit. He is taken to the operating room for surgical exploration where 1.8
cm of scar tissue between the tendon ends is identified. The tendon sheath is found to
be intact and allows smooth passage of a pediatric urethral catheter. What is the next
step in management? Review Topic

1 Resection of scar and primary repair of tendon ends.


2 Resection of scar and adjacent 1cm of tendon, placement of Hunter rod for staged
reconstruction.

3 Debulking of scar, partial excision of 25% of the A2 and A4 pulleys.

4 Resection of scar, harvest of ipsilateral palmaris longus tendon for tendon reconstruction.

5 Resection of scar and proximal tendon, tendon transfer from adjacent digit.

PREFERRED RESPONSE 4

This patient sustained an FDP laceration that was treated initially with primary repair.
He subsequently re-ruptured the tendon 2 months later. With scar >1 cm, tendon
grafting is indicated and primary tendon grafting with palmaris longus is commonly
performed as it is the most accessible tendon in the operative field.

Flexor tendon lacerations commonly result from volar lacerations. Concomitant


neurovascular injury is common. Partial lacerations <60% of tendon width are treated
with debridement and early range of motion. With partial lacerations, the least amount
of gliding resistance can be obtained with debridement alone. Lacerations >60% of
tendon width are treated with flexor tendon repair and controlled mobilization. Failed
primary repair and chronic untreated injuries are indications for flexor tendon
reconstruction and intensive postoperative rehabilitation.

Lilly et al. reviewed complications after flexor tendon injuries. Common


complications include adhesions, joint contracture, tendon rupture, triggering, pulley
failure and bowstringing, quadrigia, swan-neck deformity and lumbrical plus
deformity.

Figure A shows a zone II laceration of the left index finger FDP.


Incorrect Answers:
Answer 1: Primary repair is not indicated. Advancement of the FDP >1cm leads to
contractures and quadrigia effect.
Answer 2: As the sheath is intact and not collapsed (as evidence by easy passage of a
catheter), primary tendon grafting is preferred as it is a single-staged procedure. If the
sheath is collapsed/scarred, two-stage reconstruction is performed.
Answer 3: The scar is structurally weak and should be excised. Pulley excision or
venting is used to decrease triggering and gliding resistance. It is not indicated in this
instance.
Answer 5: Tendon transfer from a nearby digit is not indicated if there is sufficient
autograft tendon (e.g. palmaris longus) for reconstruction.

(SAE08UE.32) A 17-year-old javelin thrower reports medial-sided elbow pain and


diminished grip strength while throwing. He has decreased sensation in the little and
ring fingers of his throwing hand only while throwing. The sensory deficits resolve at
rest. Examination of the elbow reveals no instability and full motion. He has a
positive Tinel’s sign over the cubital tunnel and a positive elbow flexion test.
Radiographs are normal. What is the next most appropriate step in management?
Review Topic

1 Anterior ulnar nerve transposition

2 Cortisone injection

3 Nighttime elbow extension splinting

4 Medial collateral ligament reconstruction

5 Ulnar nerve decompression in situ

PREFERRED RESPONSE 3

The patient’s symptoms and examination findings are consistent with ulnar
neuritis/cubital tunnel syndrome, most probably exacerbated by javelin throwing. The
first step includes rest and extension splinting. Surgical intervention should only be
considered after failure of nonsurgical management.
(SAE08AN.41) A 37-year-old patient with type I diabetes mellitus has a flexor
tenosynovitis of the thumb flexor tendon sheath following a kitchen knife puncture
wound to the volar aspect of the thumb. Left unattended, this infection will likely first
spread proximally creating an abscess in which of the following spaces of the palm?
Review Topic

1 Central space

2 Hypothenar space

3 Carpal tunnel

4 Posterior adductor space

5 Thenar space

PREFERRED RESPONSE 5

Flexor tenosynovitis of the thumb flexor tendon sheath can spread proximally and
form an abscess within the thenar space of the palm. The flexor pollicis longus tendon
does not pass through the central space of the palm or the hypothenar space of the
palm. The flexor pollicis longus tendon does pass through the carpal tunnel, but this is
not a palmar space. The three palmar spaces include the hypothenar space, the thenar
space, and the central space. The posterior adductor space would likely only be
involved secondarily after spread from a thenar space infection.

(SAE09TR.89) A 66-year-old woman was a restrained passenger in an automobile


accident. She sustained a direct blow to her nondominant left hand as the airbag in her
automobile deployed and she now reports pain, swelling, and difficulty moving her
fingers. Radiographs are shown in Figures 58a and 58b. Appropriate definitive
treatment should consist of Review Topic
1 application of a compressive soft dressing, and aggressive edema control and range-of-
motion exercises.
2 plaster immobilization without reduction of the fractures.

3 closed reduction and plaster immobilization.

4 surgical fixation of the middle finger metacarpal and closed treatment of the ring and little
finger metacarpals.

5 surgical fixation of all three metacarpal fractures.

PREFERRED RESPONSE 5

While most isolated metacarpal fractures can be treated nonsurgically, multiple


metacarpal fractures are inherently unstable due to the loss of support that an intact
adjacent metacarpal provides; therefore, treatment should consist of surgical fixation
of all three metacarpal fractures.

(SAE10SM.8) A 23-year-old national team rower reports pain over the radial dorsum
of the forearm that is made worse with flexion and extension of the wrist during
competition. His primary physician initially diagnosed de Quervain's tenosynovitis,
and a subsequent corticosteroid injection into the first dorsal compartment at the wrist
provided no relief. The patient continues to report pain and audible crepitus that is
noted 5 cm proximal to the wrist joint, on the radial aspect. What structures are
involved in the continued pathology? Review Topic

1 Abductor pollicis brevis and extensor pollicis brevis

2 Abductor pollicis brevis and extensor pollicis longus

3 Abductor pollicis longus and extensor pollicis brevis

4 Abductor pollicis longus and extensor pollicis longus

5 Adductor pollicis and extensor pollicis longus


PREFERRED RESPONSE 3

Intersection syndrome is also known as "squeakers wrist," "oarsmen wrist," and


crossover tendinitis. It occurs where the first and second dorsal wrist compartment
structures pass over one another, resulting in fibrosis, muscular changes, and
inflammation of the bursa in this area. The structures involved are the abductor
pollicis longus and extensor pollicis brevis (first dorsal compartment) that pass across
the second compartment structures (extensor carpi radialis brevis and extensor carpi
radialis longus). An audible "squeak" is occasionally heard at the intersection point,
which is approximately 4 to 5 cm proximal to the proximal dorsal wrist crease.

(SAE11AN.24) A 38-year-old man reports a 6-month history of pain in his left wrist.
He denies any injury and is otherwise healthy. An MRI scan is shown in Figure 24.
What is the recommended treatment? Review Topic

1 Radial shortening osteotomy


2 Lunate excision with tendon interposition

3 Lunate implant arthroplasty

4 Ulnar shortening osteotomy

5 Total wrist arthrodesis

PREFERRED RESPONSE 1

The MRI scan shows avascularity (decreased signal intensity on T1-weighted image)
of the lunate in an ulnar minus wrist, consistent with Kienbock's disease. No
degenerative changes are seen in the carpus. Of the choices listed, radial shortening
osteotomy is the treatment of choice. This procedure provides an extra-articular
approach to treatment. The other options could be considered in more advanced cases
or if joint deterioration/destruction was noted. Lunate excision with tendon
interposition and lunate implant arthroplasty are rarely used at this time. An ulnar
shortening osteotomy could make the problem worse by increasing the contact forces
between the radius and lunate. A limited intercarpal fusion is usually used prior to
resorting to total wrist arthrodesis.

(OBQ13.37) A 35-year-old mixed martial arts fighter and recreational cocaine user
presents with symptoms concerning for hypothenar hammer syndrome (HHS).
Significant ischemia is found on physical exam. Arteriography is shown in Figure A.
What is the most appropriate next step in treatment? Review Topic

1 Conservative treatment with cocaine abstinence


2 Conservative treatment with activity modifications and medical management with calcium
channel blockers

3 Therapeutic endovascular fibrinolysis

4 Excision of involved segment and reconstruction with or without a vein graft

5 Medical management with coumadin for 6 months

PREFERRED RESPONSE 4

Figure A shows a bilobed aneurysm overlying the ulnar artery with normal appearing
distal vasculature. Hypothenar hammer syndrome (HHS) can be associated with an
aneurysm and is most appropriately treated with resection of the involved segment
and either reconstruction with a primary anastomosis or vein graft.

HHS syndrome consists of two separate entities, thrombosis and aneurysm. In the
setting of thrombosis without aneurysm, conservative management is preferred. If the
thrombosis is acute (<2 weeks), endovascular fibrinolysis has shown good results. In
patients with an HHS and an aneurysm, surgery is required for resection to prevent
distal embolization and remove the often painful aneurysmal mass.

Yuen et al. review HHS. In patients with HHS and aneurysms, resection of the
involved segment of the ulnar artery prevents distal embolic events, eliminates the
painful mass, relieves ulnar nerve compression, and removes the thrombus which
initiated the reflex vasospasm and closed off the collateral vessels in the region.

Lifchez et al. review the long-term outcomes of 11 patients with HHS treated with
ulnar artery reconstruction. 2 of the patients underwent excision and direct ulnar
artery repair, and the rest underwent reconstruction with a vein graft. All patients had
a mean improvement in digital brachial index, decrease in pain and dysesthesia
symptoms, and decrease in cold intolerance compared with preoperatively.

Nitecki et al. review a case series of 6 patients with HHS. They state that the
treatment of thrombosis should be largely conservative, but thrombolytic treatment
could be considered if the event happened <2 weeks prior to presentation.

Illustration A shows an excised ulnar artery aneurysm in a patient with HHS. Note the
typical "corkscrew" appearance of the distal segment.

Incorrect Answers:
Answer 1: HHS with aneurysm is best treated surgically.
Answer 2: HHS with aneurysm is best treated surgically.
Answer 3: Therapeutic endovascular fibrinolysis is useful for acute thrombosis
without aneurysm.
Answer 5: HHS with associated aneurysm is best treated surgically.

(SAE11AN.97) When releasing a proximal interphalangeal (PIP) joint flexion


contracture, the check rein ligaments are released first, followed by which of the
following structures? Review Topic

1 Proper collateral ligament from the proximal phalanx

2 Proper collateral ligament from the middle phalanx

3 Extensor tendon

4 Dorsal capsule

5 Accessory collateral ligament and volar plate


PREFERRED RESPONSE 5

When releasing a PIP joint flexion constracture, each step should be followed by an
attempt to extend the PIP joint. If there is no passive extension, then the next stage is
performed. The steps for a volar approach PIP flexion contracture release are as
follows: retract the flexor tendons after appropriate pulley takedown; release check
rein ligaments; then accessory collateral ligament and volar plate; and finally the
proper collateral ligament is then released off the proximal phalanx. Extensor
tenolysis only needs to be performed if there is no active extension.

(SBQ12FA.22) Which of the following options is the MOST reliable method to


reduce the complication seen in Figure A Review Topic

1 Use of hard soled shoes


2 Use of industrial toe protection footwear

3 Compression stockings

4 Thermal insulation footwear

5 High protein diet

PREFERRED RESPONSE 4

This patient presents with bilateral frostbite of the toes. The most reliable method to
reduce the risk of cold exposure injury is to reduce thermal heat loss.

Frostbite is a cold exposure injury to local tissues and can be classified as a first-,
second-, third-, and fourth-degree injury, depending on the depth of tissue damage.
Risk factors for injury include: prolonged exposure to sub-zero temperature, increased
wind chill factor, direct skin contact with wind, water or metal, poor thermal
insulation, alcohol abuse, smoking, peripheral vascular disease, diabetes, and prior
cold exposure injury.

Kuklane summarizes the research on cold protection of feet. The authors state that the
toes have little local metabolic heat production because of their small muscle mass,
and the capacity to generate heat decreases with tissue temperature. They concluded
that footwear thermal insulation is the most important factor for protection against
cold induced injury.

Figure A shows a third degree frostbite injury to the toes, bilaterally. Illustration A
shows the classification system of frostbite injury.

Incorrect Answers:
Answer 1: Comfortable shoes (not hard soled) have been shown to decrease the
incidence of frostbite
Answer 2: This is not a picture of an industrial toe crush injury.
Answer 3: Compression stockings would not reduce the risk of thermal injury to the
toes.
Answer 5: Malnourished or chronically ill are also more susceptible to cold-induced
injuries because of their decreased capacity for normal thermoregulation. However,
this is not the most reliable measure to reduce the risk of thermal injury.

(SAE08AN.20) The attachments of the transverse carpal ligament include which of


the following structures? Review Topic

1 Scaphoid and the ulna

2 Trapezium and the hook of the hamate

3 Trapezium and the triquetrum

4 Trapezoid and the hook of the hamate

5 Trapezoid and the pisiform

PREFERRED RESPONSE 2

The transverse carpal ligament is the volar boundary of the carpal tunnel. It attaches
to the scaphoid and trapezium radially and the pisiform and the hook of the hamate
ulnarly. The ulna and trapezoid do not receive attachments of the transverse carpal
ligament.
(SAE11UE.12) Figures 12a and 12b show the initial radiographs of a 27-year-old
snow boarder who fell backward onto his left outstretched hand. Which of the
following most accurately describes the sequence of events that occurred during this
injury? Review Topic

1 Lunotriquetral ligament failure followed by distal row dissociation, scaphoid extension,


scaphoid failure, and dorsal dislocation of the carpus
2 Volar dislocation of the lunate followed by scaphoid extension, scaphoid failure,
lunotriquetral failure, and distal row dissociation

3 Dorsal intercarpal ligament failure followed by distal row dissociation, scaphoid failure,
lunotriquetral ligament failure, and dorsal dislocation of the carpus

4 Short radiolunate ligament failure followed by volar dislocation of the lunate,


lunotriquetral ligament failure, scaphoid failure, and distal row dissociation

5 Scaphoid extension followed by scaphoid failure, distal row dissociation, lunotriquetral


ligament failure, and dorsal dislocation of the carpus

PREFERRED RESPONSE 5

As described by Mayfield and associates, the typical sequence of events referred to as


"progressive perilunar instability" that result in a volar perilunate dislocation are as
follows: scaphoid extension, followed by opening of the space of Poirer, scaphoid
failure, and distal row dissociation, which in turn lead to hyperextension of the
triquetrum, lunotriquetral ligament failure, and finally dorsal dislocation of the carpus.
The lunate remains in the lunate fossa in a perilunate fracture-dislocation but is
dislocated in a lunate dislocation. The short radiolunate and dorsal intercarpal
ligaments typically remain intact.
(OBQ15.15) A 27-year old female sustains the closed injury shown in Figures A and
B. She is treated non-operatively with splint immobilization. At follow-up, she is
found to have an extension lag at the metacarpophalangeal (MCP) joint along with
reduced grip strength. Which finding is the most significant contributor to this
outcome? Review Topic

1 Rotational deformity
2 15-degree apex-dorsal angulation

3 Nonunion

4 5mm of shortening

5 Extensor tendonitis

PREFERRED RESPONSE 4

Shortening of metacarpal fractures >2-5mm may result in extension lag at the MCP
joint as well as reduced grip strength due to loss of tension on the extensor
mechanism.

Indications for nonoperative treatment of metacarpal shaft fractures with splint or cast
immobilization include stable fracture pattern, no rotational deformity, and acceptable
angulation. The amount of acceptable angulation differs for each metacarpal: 10-20
degrees for index and long fingers, 30 degrees for ring fingers, and 40 degrees for
small fingers.

Strauch et al. performed a cadaveric study quantifying the magnitude of MCP


extensor lag produced by metacarpal shortening. The authors found that every 2mm
of metacarpal shortening resulted in an average of 7 degrees of extensor lag at the
MCP joint.
Al-Qattan et al. prospectively reviewed conservative management of spiral/long
oblique fractures of the metacarpal shaft treated with palmar wrist splinting.
Extension lag was initially seen in all digits, but recovered by 1 year. Grip strength at
1 year was also found to be 94% of the contralateral hand. The authors' indications for
surgery included severe angulation with failure of closed reduction, rotational
malalignment, and significant displacement with loss of bone-to-bone contact.

Figures A and B demonstrate AP and lateral views of a shortened, oblique fracture of


the fourth metacarpal shaft.

Incorrect Answers:
Answer 1: Any amount of rotational deformity is an indication for surgical
intervention, but rotation would not result in an extension lag.
Answer 2: Angulation may result in hyperextension of the MCP and an extension lag
of the PIP.
Answer 3 and 5: Nonunion and tendonitis are rare and would not result in an
extension lag.

(OBQ15.263) A 23-year-old football player falls on his right hand during a game.
Figure A is an injury radiograph of his right hand. Figure B shows the main
ligamentous restraints, ligaments 1 through 4 (L1 through L4). Which of the
following is the MOST LIKELY scenario?
Review Topic

1 This is most likely a volar dislocation. L2 is torn.


2 This is most likely a volar dislocation. L3 is torn.

3 This is most likely a dorsal dislocation. L2 is torn.

4 This is most likely a dorsal dislocation. L3 is torn.

5 This is most likely a dorsal dislocation. Both L2 and L3 are torn.


PREFERRED RESPONSE 4

This is a dorsal dislocation of the thumb CMC joint. The dorsoradial ligament (DRL,
L3) is torn. The anterior oblique ligament (AOL, L2) remains intact but peels off the
base of the 1st metacarpal (MC).

Thumb CMC dislocation is rare. Dorsal dislocations are more common than volar
dislocations. The mechanism for dorsal dislocation is axial loading with thumb MC
flexed. (Another possible mechanism is a force driven into the 1st web e.g. handlebar
driven into motorcyclist’s thumb on impact). The primary restraint to dorsal
dislocation is the DRL. The AOL remains continuous is but peeled off the 1st
metacarpal base. In contrast, in a Bennett/Rolando fracture, the AOL remains attached
to a fragment of bone at the MC base and the MC translates because of the fracture
through the base.

Strauch et al. reviewed acute CMC dislocations in cadavers. They applied a dorsal
dislocating force and progressively sectioned ligaments. They found that the primary
restraint was the DRL with the AOL allowing dislocation after peeling off the base of
the 1st metacarpal. The joint was more stable in pronation and extension which
tightened the anterior oblique ligament.

Bettinger and Berger reviewed the functional anatomy of the trapezium and TMCJ.
They describe 16 ligaments that help stabilize the CMCJ and use geometric and
mathematical models to describe and support ligament function. They state that the
dorsoradial ligament is an important stabilizer against dorsal forces. The
trapeziotrapezoid, trapezio-II metacarpal and trapezio-III metacarpal ligaments
function as tension bands to support the trapezium against cantilever bending forces.

Figure A shows a dorsal dislocation of the thumb CMC joint. Figure B shows the
main ligamentous restraints of the CMC joint. L1 is the intermetacarpal ligament, L2
is the AOL, L3 is the DRL, L4 is the posterior oblique ligament. Illustration A shows
dislocation of a cadaver thumb. Panel A shows the AOL (black arrow) and DRL
(white arrow). With sectioning of the DRL (Panel B) and a dorsal directed force, the
AOL strips of the 1st MC base (Panels C and D). Illustration B shows how both DRL
and AOL are of equal laxity when reduced, but with dorsal subluxation, the DRL
tightens and the AOL remains lax. Of note, the MC base is 34% wider than the
trapezium (represented by spheres of differing sizes), which also contributes to
differential laxity with dorsal translation.

Incorrect Answers:
Answers 1 and 2: This is a dorsal dislocation
Answers 3 and 5: The AOL (L3) remains in continuity but is peeled/stripped off the
1st MC base.
(SAE10PE.20) Pediatric flexor tendon injuries of the upper extremity differ from
adult flexor tendon injuries in which of the following ways? Review Topic

1 Delayed presentation is not common.

2 A staged repair is never necessary.

3 Six to eight weeks of postoperative immobilization is recommended.

4 Cooperation with occupational therapy can be difficult.

5 The use of Botulinum is contraindicated.

PREFERRED RESPONSE 4

Pediatric flexor tendon injuries have several remarkable distinctions from those in
adults. Delayed presentation is more common in children, at times requiring staged
flexor tendon reconstruction. Three to four weeks of postoperative immobilization
following acute repair is recommended in children as opposed to early motion
protocols used in adults. Temporary paralytic agents (botulinum toxin type A) have
also been shown to facilitate the rehabilitation phase of flexor tendon care in very
young children.

(SAE11OS.139) Which of the following substances is likely to cause the most soft-
tissue damage in the long term if injected into a fingertip under high pressure? Review
Topic

1 Grease

2 Latex paint

3 Water

4 Oil-based paint

5 Chlorofluorocarbon-based refrigerant
PREFERRED RESPONSE 4

This type of injury represents a difficult problem in hand surgery. The factors that
most determine outcome after high-pressure injection injuries into the fingertip
include: involvement of the tendon sheath, extent of proximal spread of the injected
substance, pressure setting, and delay to surgical treatment. The other factor that
likely is most important is the type of substance injected. Water and latex-based paints
are least destructive. Grease and chlorofluorocarbon-based substances are
intermediate, but aggressive surgical debridement can restore reasonable function.
Oil-based paints are highly inflammatory and can cause such chronic inflammation
such that amputation may be the only reasonable treatment option despite early
aggressive surgical treatment.

(SAE11UE.114) What additional procedure should be done when performing a


radioscapholunate fusion for posttraumatic arthrosis following a distal radius fracture?
Review Topic

1 Excision of the triquetrum and distal pole of the scaphoid

2 Anterior interosseous neurectomy

3 Fascial interposition arthroplasty of the capitolunate joint

4 Sectioning of the dorsal intercarpal ligament

5 Ulnar shortening osteotomy

PREFERRED RESPONSE 1

Excision of the triquetrum and distal pole of the scaphoid frees up the mid-carpal
joint, improving radial deviation and the flexion-extension arc of motion of the wrist.
This offers an alternative to complete wrist arthrodesis for posttraumatic arthrosis of
the radiocarpal joint. An anterior interosseous neurectomy is believed to decrease
some pain transmission from the wrist but because the fusion is done dorsal, cutting
this volar structure is not routinely done. Fascial interposition is not needed because
the capitolunate should be preserved in posttraumatic radiocarpal arthrosis. Sectioning
of the dorsal intercarpal ligament would provide no benefit. If the triquetrum is
excised, then an ulnar shortening osteotomy is unnecessary.
(OBQ15.96) A 21-year-old male underwent percutaneous screw fixation of a
nondisplaced scaphoid waist fracture 7 months ago and now presents with persistent
radial sided wrist pain. His skin wounds are well-healed. Basic radiographs of the
wrist show maintained hardware position. What is the next best step? Review Topic

1 Clinched fist stress radiograph

2 Bone scan

3 MR arthrography

4 Fat-suppressed, T2-weighted MR Imaging

5 Fine cut CT scan

PREFERRED RESPONSE 5

After percutaneous screw fixation of a nondisplaced scaphoid waist fracture, the best
imaging study to assess for union is fine cut CT scan of the scaphoid along the
scaphoid axis.

The etiology of chronic pain symptoms after dorsal percutaneous screw fixation of
scaphoid waist fractures include fracture non-union, fracture pseudoarthrosis, loose,
migrated, and/or broken implants, as well as extra-osseous factors (e.g. neuroma).
Fine cut CT scans have shown to be the most specific imaging study to assess for
fracture union, after technical errors have been excluded with basic radiographs (eg,
inadequate screw size or poor screw position).

Ring et al. reviewed acute scaphoid fractures. They state that surgical treatment is
recommended for displaced and complex fractures (open fractures, perilunate
fracture-dislocations, and scaphoid fractures associated with fracture of the distal
radius), very proximal fractures, and fractures for which the diagnosis and treatment
have been delayed.

Lutsky et al. suggest that scaphoid nonunion is conventionally defined by no healing


of the fracture greater than 6 months after fixation of the injury. They state that CT
scans are most useful for diagnosing scaphoid union, but they are unreliable for ruling
out nonunion.

Yin et al. examined the cost effectiveness of multiple competing diagnostic strategies
for suspected scaphoid fractures. They state that immediate CT and MRI were the
most cost-effective strategies for diagnosing suspected scaphoid fractures.

Illustration A shows a persistent fracture line with CT imaging. Illustration B shows


axial view of the scaphoid with distal cortical hardware perforation of the screw.

Incorrect Answers:
Answer 1: Clinched fist stress radiographs assess for scapholunate dissociation and
SL ligament injury.
Answer 2: Bone scans are non-specific studies. It may show increased signal at the
scaphoid, but this would not be specific for fracture union/non-union in the early time
period.
Answer 3-4: The amount of metal from MR imaging would not provide reliable
findings of fracture union. However, MRI is considered the best imaging study to
assess for nondisplaced, acute fractures when radiographs and CT images are
negative.

(SAE11PA.77) A 45-year-old woman has a painful mass in the dorsum of the right
wrist. It is firm and nontender to palpation. She states it has slowly gotten bigger over
the past 3 years. You suspect a dorsal wrist ganglion. What is the most definitive way
to confirm this diagnosis? Review Topic

1 Observe it for 1 year to see if it changes dramatically in size.

2 Obtain a gadolinium enhanced MRI scan.

3 Obtain radiographs, looking for scapholunate joint degenerative changes.

4 Perform a needle aspiration and send the aspirate for cytologic examination.

5 Apply direct firm manual pressure over the mass to see if it can be ruptured.

PREFERRED RESPONSE 4

Dorsal wrist ganglions are synovial cysts that arise most frequently from the
scapholunate joint. They often extend between the extensor digitorum communis and
extensor pollicis longus tendons at the wrist. Aspiration of the cyst is both
oncologically safe if done appropriately and also the easiest way to definitively
confirm the diagnosis. Clear, yellow viscous fluid/gel is most often aspirated.
Cytologic evaluation is mandatory to exclude myxoid neoplasms. Because the lesion
has been present for 3 years, further observation is not warranted. The classic
presentation, physical examination findings, and location make MRI and radiographs
unnecessary. Manual rupture of the mass is not recommended.
(SAE11UE.50) What is the effect of shortening of metacarpal fractures? Review
Topic

1 Causes the greatest degree of extensor lag in the index finger

2 Causes the greatest degree of extensor lag in the little finger

3 Results in an average extensor lag of 7 degrees for every 2 mm of shortening

4 Results in an average extensor lag of 14 degrees for every 2 mm of shortening

5 Has no effect on grip strength

PREFERRED RESPONSE 3

Cadaveric models have demonstrated a 7-degree extensor lag for every 2 mm of


metacarpal shortening, with the amount of lag increasing in a linear fashion. There
was no statistical difference in the amount of lag in regard to the digit involved. Based
on muscle length-tension relationships, cadaveric models have also been used to
demonstrate an 8% loss of power secondary to decreased interosseous force
generation with 2 mm of shortening. Because the intrinsic muscles of the hand
contribute anywhere from 40% to 90% of grip strength, decreased interosseous force
generation secondary to metacarpal shortening will invariably cause a decrease in grip
strength.

(SAE11AN.95) What is the most common site of posterior interosseous nerve


compression in radial tunnel syndrome? Review Topic

1 Fibrous bands superficial to the radiocapitellar joint

2 Radial recurrent artery branches (leash of Henry)

3 Fibrous edge of the supinator (arcade of Frohse)

4 Distal edge of the supinator

5 Tendinous margin of the extensor carpi radialis brevis (ECRB)


PREFERRED RESPONSE 3

The five compression sites described in radial tunnel syndrome are: the distal edge of
the supinator; fibrous bands superficial to the radiocapitellar joint; tendinous margin
of the extensor carpi radialis brevis (ECRB); radial recurrent artery (leash of Henry);
and the most common site of compression, the fibrous edge of the supinator (arcade
of Frohse). The tendinous portion of the supinator is next to the bone and does not
compress the posterior interosseous nerve.

(OBQ13.9) Figure A shows a traumatic laceration of the distal forearm with a 5cm
segmental median nerve defect. Which of the following repair or reconstruction
techniques would allow for the best recovery of motor function? Review Topic

1 Autogenous venous nerve conduit


2 Collegen synthetic nerve conduit

3 Biodegradable polyglycolic acid

4 Processed nerve allograft

5 Nerve autograft

PREFERRED RESPONSE 5

Figure A shows a traumatic laceration with 5cm of median nerve defect. The use of
nerve autograft for this size defect has been shown to have the best recovery of motor
function.

The optimal surgical treatment of nerve laceration is direct tension-free repair. In


segmental nerve defects this approach cannot be achieved. The use of interposed
autologous nerve grafting remains the gold standard of repair in this setting. The use
of alternative techniques, such as processed allografts and synthetic conduits, have not
shown to have equivalent recovery of motor function as compared to nerve autograft.

Giusti et al. used a rat model to examine techniques of peripheral nerve repair. They
showed that nerve autograft resulted in better motor recovery than did the use of
processed allograft or a collagen conduit.

Deal et al. discussed tubular interposition substitutes, or nerve conduits, as an


alternative to nerve autograft in segmental nerve defect. Nerve conduits can include
autogenous nerve conduits (venous or arterial) and synthetic nerve conduits (collagen,
PGA, or caprolactone). In general, there is an upper limit of 3-cm when using nerve
conduit.

Figure A is an image of the volar forearm. There is a traumatic laceration to the


anterior compartment tendons as well as the median nerve.

Incorrect Answers:
Answer 1: Autogenous venous nerve conduit has been shown to effective in digital
nerve defects. However, the results are not superior to nerve auto grafting and have
not been shown to be effective in gaps >5-cm.
Answers 2,3: Synthetic nerve conduit (eg. collagen, biodegradable polyglycolic acid)
have not been shown to be superior to nerve auto grafting. Synthetic nerve conduit
may be useful in digital nerves with gaps <3-cm.
Answer 4.: Processed allograft has been shown to be inferior to nerve autograft in
terms of motor recovery.

(SAE11UE.86) A 44-year-old woman with cubital tunnel syndrome and associated


ulnar nerve subluxation with elbow flexion has failed to respond to nonsurgical
management. Which of the following statements is most acccurate regarding in situ
simple decompression of the nerve compared with subcutaneous anterior
transposition? Review Topic

1 Patients undergoing anterior transposition have improved motor outcomes.

2 Patients undergoing anterior transposition have improved sensory outcomes

3 Patients undergoing simple decompression have improved motor outcomes.

4 Patients undergoing simple decompression have improved sensory outcomes.

5 No differences in outcome are likely between treatment types.


PREFERRED RESPONSE 5

Recent reports comparing outcomes of surgical treatment of ulnar nerve compression


at the elbow have demonstrated no differences in outcome between simple
decompression and anterior transposition. The presence of subluxation of the ulnar
nerve was not a contraindication to in situ decompression in the study by Keiner and
associates.

(OBQ14.28) A 25-year-old man sustains a shearing injury to the dorsal aspect of his
ring finger. The nail plate was avulsed, leaving the nail bed exposed. The eponychial
fold was also avulsed, leaving terminal extensor tendon exposed, devoid of paratenon.
A clinical photograph is seen in Figure A. What surgical procedure will provide the
best soft tissue coverage? Review Topic

1 Full-thickness skin graft


2 Toe nail bed graft

3 Volar cross-finger flap

4 Heterodigital, reversed neurovascular island flap

5 Reversed cross finger flap

PREFERRED RESPONSE 5

This patient has a dorsal digital defect with exposed tendon. A reversed cross finger
flap (RXFF) from the adjacent finger (middle finger) will provide the best soft tissue
coverage.
A RXFF is prepared similar to a standard cross finger flap (XFF) except that the hinge
and donor tissue differ. The hinge of the flap is opposite the primary defect, unlike a
standard XFF, where the hinge is adjacent to the defect. Dissection is kept to the level
of the dermis, below hair follicles but above the subcutaneous veins. The
subcutaneous tissue is raised and inset into the defect, leaving the donor paratenon
untouched. A full thickness skin graft is then set on the flap. Flap division occurs at 2-
3weeks.

Kappel et al. describe the XFF in 205 patients. The index finger was the most
common recipient, and the middle finger was the most common donor finger. Half
had cold intolerance and mean 2 point discrimination was 8.25mm. Possible
complications include hyperpigmentation and finger stiffness (especially in older
patients). Contraindications include vasospastic conditions (e.g. Buergers) and
preexisting deformity (e.g. Dupuytrens).

Atasoy described the technique of a RXFF. He states that the best donor site is the
dorsum of the middle and proximal phalanges, and recommends avoiding the dorsum
of DIP and PIP joints because of thinness of subcutaneous tissues in these regions.

Figure A shows a dorsal digital defect with exposed extensor tendon. Illustration A
shows the steps in harvesting and insetting a reversed cross finger flap. Illustration B
shows the difference between standard (above) and volar (below) cross finger flaps.
Illustration C shows a volar cross finger flap from the middle finger. Illustration D
shows a neurovascular island flap.

Incorrect Answers
Answer 1: Direct application of a full thickness skin graft will lead to adhesions to
exposed tendon that is devoid of paratenon, and interfere with tendon gliding. An
intermediary layer of subcutaneous tissue (from a RXFF) is necessary.
Answer 2: A toe nail bed graft is used for nail bed defects and will not address
eponychial deficiency.
Answer 3: A volar cross finger flap (uncommon) is used to close a distal thumb
amputations. The blood supply of the volar flap is vertical, limiting the size of flap
that can be elevated without necrosis. The pedicle of the volar XFF must not be
dissected free of the digital neurovascular pedicle near its base. When dividing the
pedicle, care must be taken not to injure the neurovascular bundles.
Answer 4: Heterodigital, reversed neurovascular island flaps are useful for coverage
of extensive defects of the fingers that cannot be reconstructed using other flaps, and
as an alternative to microsurgical reconstruction.
(OBQ15.103) Closed rupture of the flexor tendons of the small finger has been
reported secondary to: Review Topic

1 Internal fixation of a distal radius fracture with a volar plate

2 Closed treatment of a distal radius fracture

3 A large volar wrist ganglion

4 Spurring of the scaphoid

5 Hook of hamate nonunion

PREFERRED RESPONSE 5

Nonunion of a hook of hamate fracture has been shown to lead to rupture of the flexor
tendons to the small finger.

Displaced hamate hook fractures are optimally treated with early excision to avoid
sequelae such as flexor tendon and nerve injury, and to allow early return to activity.
Nonunions are often a result of missed injuries, failure of nonoperative treatment, or
failure of attempted ORIF. The hook acts as a fulcrum for the ulnar digital flexor
tendons and it is believed that excision of the hook may lead to loss of grip strength,
though this remains controversial. The proximity of the flexor tendons to the hook
may also explain the reported occurrence of closed ruptures of these tendons in the
presence of a nonunion of the hook.

Yamazaki et al. present a case series of 6 patients with hook of hamate fractures that
went on to nonunion and subsequently suffered closed ruptures of the flexor tendons
to the small finger. They compared morphology of the fractures and found even small,
rounded fracture fragments can lead to tendon rupture.

Klausmeyer et al. provide a review of the treatment of hook of hamate fractures. They
note that treatment of a hook of hamate fracture or nonunion is based on a reported
11% loss of flexor tendon excursion and case series of athletes who could not return
to sport following closed treatment alone. Published series of open reduction internal
fixation or excision of hook of hamate fractures both showed good results and return
to sport.

Illustration A is a CT image of a nonunion of a hook of hamate fracture. Illustration B


is an intraoperative photograph of a ruptured flexor tendon about a hook of hamate
fracture nonunion.

Incorrect answers:
Answer 1: Internal fixation of a distal radius fracture with a volar plate has been
shown to potentially lead to rupture of the flexor pollicis longus, particularly when the
plate placed distal to the watershed line.
Answer 2: Closed treatment of distal radius fractures has been shown to potentially
lead to closed rupture of the extensor pollicis longus tendon.
Answer 3: Volar wrist ganglions are not known to lead to closed rupture of any flexor
tendons.
Answer 4: Spurring of the scaphoid has been associated with rupture of the radial
flexor tendons

(OBQ14.103) All of following are absolute indication to replantation in the hand


EXCEPT?

Review Topic

1 Amputation of multiple digits

2 Thumb amputation

3 Amputation at the level of the palm

4 Single digit amputation distal to the flexor digitorum superficialis insertion

5 Isolated amputation of little finger proximal to flexor digitorum superficialis in a child

PREFERRED RESPONSE 4

A single digit amputation distal to the flexor digitorum superficialis (FDS) insertion is
a RELATIVE indication for replantation, NOT an absolute indication.

Absolute indications for replantation include thumb amputations, multiple digit


amputations, amputations at the level of the wrist or palm and any amputation in a
child. The aforementioned injuries result in significant loss of hand function and are
therefore a priority for surgical repair. Although replantation of single digit
amputations is rarely indicated, amputations distal to the FDS insertion have been
found to have good functional results after replantation. As such, these injuries are a
relative indication for replantation.

Soucacos reviewed the indications and selection criteria for digital amputation and
replantation. He found the most significant guideline underlining the philosophy of
digital replantation to be the aim of not only ensuring the survival of a digit, but its
functional use.

Friedrich et al. analyzed the geographic distribution of upper extremity replant


procedures, patient factors, and characterized the facilities performing these
procedures. They found that regionalization of replantation care and, more broadly,
emergency hand care may allow the hand surgery community to provide more
efficient and facile care for these patients.

Illustration A shows an amputation of the index finger distal to the FDS insertion.
Figure B and C are clinical and radiographic images, respectively, that demonstrate
the successful replantation of the amputated digit.

Incorrect Answers:
Answers 1, 2, 3 and 5: These are all absolute indications for replantation.

(SAE11UE.3) Figure 3 shows an arthroscopic view of the radiocarpal joint from the
3-4 portal, looking volarly and radially (Sc=scaphoid, R=Radius). What structure is
marked by the asterisk? Review Topic

1 Radioscaphocapitate ligament
2 Scapholunate ligament

3 Palmar oblique ligament

4 Dorsal intercarpal ligament

5 Triangular fibrocartilage complex (TFCC)

PREFERRED RESPONSE 1

The radioscaphocapitate ligament is a volar capsular structure running obliquely from


the radial styloid to the scaphoid waist, ultimately inserting on the proximal radial
aspect of the capitate. The radioscaphocapitate ligament is important in preventing
ulnar translocation of the carpus. The scapholunate ligament is located intra-
articularly, between the scaphoid and lunate. The dorsal intercarpal ligament is a
dorsal structure, and not visible during routine wrist arthroscopy. The palmar oblique
ligament connects the first and second metacarpal bases. The TFCC is visible during
wrist arthroscopy between the radius and ulna.

(OBQ14.140) A 36-year-old man suffered a zone II complete flexor pollicis longus


laceration 3 months ago, which was treated with primary repair. He was subsequently
lost to follow-up. Now he has no active thumb flexion and imaging studies show a
failure of the repair. The decision is made to proceed with a two-stage procedure.
Which of the following statements is TRUE regarding this treatment approach?
Review Topic

1 Palmaris brevis is the most common autograft used

2 Postoperative tenolysis is required in less than 30% of cases

3 Supple skin, a sensate digit, and adequate vascularity are required

4 Preserved tendon sheaths are required for two-stage repair

5 Full range of motion of adjacent joints is not necessary

PREFERRED RESPONSE 3

Supple skin, a sensate digit, and adequate vascularity are required for a two stage
procedure. Given the failure of primary repair and chronicity of the injury, flexor
pollicis longus two stage reconstruction is a reasonable method of treatment.

Flexor pollicis longus injuries can be classified into 3 zones (zone I: distal to IP joint,
zone II: between IP and MCP joints, and zone III: deep to thenar muscles). Early end
to end tendon repair is the treatment of choice but when primary repair fails and/or
injury is chronic, reconstruction is advocated. Majority of reconstructions involve a
first stage with insertion of a temporary silicone rod and subsequent return at least 3
months later for removal of the rod and insertion of autograft. In few cases, one stage
reconstruction may be indicated. Alternatively, arthrodesis of the thumb IP joint may
also be considered in this scenario to stabilize the thumb in slight flexion, as this
procedure has quicker return to function and less complications than reconstruction.

Kutsumi et al. evaluated gliding resistance at the edge of the oblique pulley in 10
cadaver thumbs. They found a significant increase in gliding resistance at the pulley
edge in thumbs undergoing tendon repair. Squaring off the oblique pulley in thumbs
undergoing repair did not decrease resistance.

Unglaub et al. reported the results of the two-stage flexor pollicis longus
reconstruction (13 patients with palmaris longus, 2 patients with plantaris tendon, one
patient with half of FCR tendon). Adequate function was regained in 75% of cases
and median DASH score was 11.

Incorrect Answers:
Answer 1: Palmaris longus is the most common autograft used
Answer 2: Postoperative tenolysis is required in more than 50% of cases
Answer 4: Preserved tendon sheaths are not required for two-stage repair as
implantation of silastic implants during the first stage will help create room for the
tendon. Preserved tendon sheaths are required in one stage reconstructions
Answer 5: Full range of motion at adjacent joints is needed for reconstruction

(SAE11UE.76) A 7-year-old boy is referred to your office 3 months after jamming his
finger while playing basketball. Examination reveals 40 degrees of active and passive
motion at the proximal interphalangeal (PIP) joint. The PIP joint is stable to radial and
ulnar stressing. Radiographs are shown in Figures 76a and 76b. What is the most
appropriate management? Review Topic

1 Observation
2 Corrective osteotomy

3 Ostectomy

4 Hand therapy for aggressive stretching

5 Dynamic splinting
PREFERRED RESPONSE 3

The most appropriate management is an ostectomy, or resection of the bone in the


subcondylar fossa region. This is a malunion where the subcondylar fossa is blocked
by malaligned bone. Because it is a bony block to motion, stretching or dynamic
splinting will be of no benefit. The physis of the proximal phalanx is proximal,
making remodeling of a fracture at the distal end very unlikely. A corrective
osteotomy has a risk of osteonecrosis of the very small distal fragment.

(OBQ12.158) A 58-year-old man presents with right middle finger swelling.


Radiographs and an MRI of his hand are shown in Figure A and B. Tissue biopsy and
staging investigations are performed. The tumor is then resected en bloc with the
middle metacarpal, which is amputated 1.5cm from the carpometacarpal joint. The
attached deep transverse intermetacarpal ligaments are sacrificed. To prevent
scissoring of the remaining digits and small objects falling through the gap between
index and ring fingers, which of the following procedures should be performed?
Review Topic

1 iliac crest bone grafting


2 ring metacarpal transposition

3 second toe transfer

4 index metacarpal transposition

5 suture of deep transverse intermetacarpal ligaments

PREFERRED RESPONSE 4

Index metacarpal transposition is indicated to reduce the space left between the index
and ring finger. In this case, the middle ray is amputated because of malignancy.
With amputation of the middle or ring metacarpals, small objects fall through the gap
and the adjacent fingers scissor. For single central ray defects, techniques to reduce
the gap include transposition of the index finger (for middle ray amputation), small
finger (for ring ray amputation), complete removal of the metacarpal (without leaving
a proximal metacarpal base stump) to allow the bases of index and ring metacarpals to
migrate together and reconstruction of the deep transverse metacarpal ligament. The
technique of index transposition may vary depending on the osteotomy (straight vs
step-cut) and fixation (K wires vs plate) as seen in the illustrations below.

Muramatsu et al. describe bony transposition for reconstruction after ray amputation
for malignancy. The advantage is immediate closure of the space. The disadvantages
include prolonged postoperative immobilization until union, malrotation (leading to
scissoring), mal-tension of tendon (because of different metacarpal heights), and
delayed or nonunion.

Lyall et al. advocate total middle ray amputation. They believe that leaving the
metacarpal base behind leads to difficulty in aligning the adjacent rays as the index
and ring must angulate over the bony obstruction to close the distal gap, leading to
scissoring. They believe that index transposition leaves an abnormally wide 1st web
space and a remnant 2nd metacarpal stump that can protrude dorsally.

Figure A is an AP radiograph of the right hand showing a destructive lesion of the


proximal phalanx of the middle finger abutting the metacarpophalageal joint. Figure B
is a STIR coronal MRI image showing the tumor mass extending into surround soft
tissue. Illustration A is a diagram showing index transposition for middle ray
amputation using a straight osteotomy and crossed K-wires. Illustration B is a
diagram showing index transposition using a step-cut osteotomy and multiple K-wire
fixation to the adjacent metacarpals. Illustration C is a diagram showing index
transposition using a straight osteotomy and plate fixation. Illustration D is a diagram
showing an alternative technique of suturing deep transverse metacarpal ligaments
together to close the gap.

Incorrect Answers
Answer 1: Iliac crest bone grafting is not indicated. Together with a skin flap, iliac
crest bone grafting may be used for thumb reconstruction.
Answer 2: Ring metacarpal transposition is not indicated. Transposing a metacarpal
from ring-to-long position leads to a new gap where the ring digit used to be. For ring
metacarpal defects, the adjacent small metacarpal is transposed instead.
Answer 3: Second toe transfer may be indicated for thumb reconstruction, or digital
reconstruction for multiple digit losses. It involves microsurgical repair. For solitary
central ray loss, a lesser procedure is preferred.
Answer 5: Gap closure by suturing deep transverse metacarpal ligaments together is
only possible in the absence of malignancy, where the ligaments are not
compromised. In this case, the ligaments were resected together with the tumor.
(OBQ15.48) A 26-year-old male sustains the index finger injury shown in Figure A.
There is loss of the sterile nail matrix with exposed bare bone. Reconstruction is best
achieved with a reverse cross finger flap from
Review Topic

1 the dorsum of the distal phalanx of the thumb


2 the dorsum of the distal interphalangeal joint of the thumb

3 the dorsum of the distal phalanx of the middle finger

4 the dorsum of the distal interphalangeal joint of the middle finger

5 the dorsum of the middle phalanx of the middle finger

PREFERRED RESPONSE 5

The injury is best resurfaced with a reverse cross finger (RXFF) flap from the dorsum
of the middle phalanx of the middle finger.

The RXFF is indicated for reconstruction of: (1) eponychial skinfold and coverage of
an exposed extensor tendon near the IP joint, (2) sterile matrix nailbed defects with
exposed distal phalanx, (3) contused, repaired, or grafted extensor tendon denuded of
paratenon, (4) boutonniere deformity with poor-quality skin over the PIP joint after
burn/avulsion injury, (5) complete avulsion of the nailbed, germinal matrix, and
surrounding skin of digits. In such an injury of the index finger, the alternative
procedure is a cross thumb to index flap, (6) elective correction of digital deformity.
Ideal donor areas include the dorsal aspect of the middle and proximal phalanges of
the adjacent fingers.

Atasoy (2016) revisits the RXFF. He describes cases of RXFF for eponychial loss,
thumb reconstruction (resection of duplication), nailbed defect, exposed bare tendon
laceration.

Atasoy (1982) described the original technique for RXFF. He states that the best
donor site is the dorsum of the middle and proximal phalanges, and recommends
avoiding the dorsum of DIP and PIP joints because of thinness of subcutaneous
tissues in these regions.
Figure A shows full-thickness tissue loss from the nailbed including the sterile matrix,
with exposed bone. Illustrations A and B shows resurfacing of the defect in Fig A
with a RXFF from the dorsum of the middle phalanx of the middle finger.

Incorrect Answers
Answers 1 and 3: There is insufficient tissue over the distal phalanges of adjacent
digits to allow creation of a RXFF large enough to cover the defects. In addition,
using tissue from this region would lead to mobilization of the eponychium and leave
the germinal matrix of the donor digit prone to injury.
Answers 2 and 4: The dorsum of the interphalangeal joints should be avoided because
of thin skin/subcutaneous tissue.

(SAE08UE.71) What is the most common bacteria cultured from dog and cat bites to
the upper extremity? Review Topic

1 Pasteurella

2 Streptococcus

3 Staphylococcus

4 Bacteroides

5 Moraxella

PREFERRED RESPONSE 1

To define bacteria responsible for dog and cat bite infections, a prospective study
yielded a median of five bacterial isolates per culture. Pasteurella is most common
from both dog bites (50%) and cat bites (75%). Pasteurella canis was the most
frequent pathogen of dog bites, and Pasteurella multocida was the most common
isolate of cat bites. Other common aerobes included streptococci, staphylococci,
moraxella, and neisseria.

(SAE11UE.5) A 22-year-old man reports a 2-week history of a burning pain along the
dorsoradial aspect of the distal forearm. The pain radiates to the dorsum of the thumb.
Examination reveals tenderness and reproduction of symptoms with percussion 8 cm
proximal to the radial styloid. Reproduction of symptoms also occurs with forearm
pronation and ulnar deviation of the wrist. No discrete sensory deficit is noted and
electrodiagnostic studies are normal. Nonsurgical management consisting of rest,
splinting, and anti-inflammatory medications for 6 weeks has failed to provide relief.
Treatment should now consist of decompression of the Review Topic

1 lateral antebrachial cutaneous nerve in the interval between the abductor pollicis longus
and the extensor pollicis brevis in the forearm.

2 lateral antebrachial cutaneous nerve in the interval between the brachioradialis and the
extensor carpi radialis longus in the distal forearm.

3 radial sensory nerve in the interval between the extensor carpi radialis longus and the
extensor carpi radialis brevis in the distal forearm.

4 radial sensory nerve in the interval between the brachioradialis and the extensor carpi
radialis longus in the distal forearm.

5 radial sensory nerve in the interval between the brachioradialis and the extensor carpi
radialis brevis in the distal forearm.

PREFERRED RESPONSE 4

Wartenberg's syndrome, or compression of the sensory branch of the radial nerve,


occurs in the interval between the brachioradialis and the extensor carpi radialis
longus approximately 8 cm proximal to the radial styloid. There may be history of
repetitive wrist/forearm circumduction activity (ie, knitting) or of wearing a tight
wristwatch or jewelry. It can occur in patients who have been handcuffed. Typical
clinical findings are pain, paresthesia, and/or hypesthesia in the dorsoradial aspect of
the wrist and hand in the distribution of the radial sensory nerve. There is often a
positive Tinel's sign over the compression site. Hypesthesia may be present in the
distribution of the radial sensory nerve which is typically on the dorsal aspect of the
first dorsal web space and dorsum of the thumb; however, with overlap in the
distribution of the superficial radial nerve and the lateral cutaneous nerve of the
forearm this may not always be present. Surgical management consists of release of
the nerve as it exits the interval between the brachioradialis and the extensor carpi
radialis longus in the distal forearm.

(SAE09TR.6) A 52-year-old woman slips in her bathroom and strikes her right hand
on a cabinet. She notes swelling, ecchymosis, and pain with attempted motion. There
are no open wounds. Radiographs are shown in Figures 5a through 5c. What is the
most appropriate treatment? Review Topic
1 Immobilization of the hand with the metacarpophalangeal (MCP) joints in flexion
2 Immobilization of the hand with the MCP joints in extension and the IP joints in extension

3 Percutaneous pinning of the proximal phalanx

4 Open reduction and internal fixation of the proximal phalanx

5 Early motion and pain management

PREFERRED RESPONSE 1

Nondisplaced transverse fractures of the phalanges are stable. Immobilization in the


intrinsic plus position will prevent MCP joint stiffness. Displaced oblique fractures
are more at risk for instability.

(SAE08OS.53) Figure 14a shows the radiograph of a 19-year-old man who hurt his
little finger while falling off his bicycle. A closed reduction of the proximal
interphalangeal (PIP) joint is performed in the emergency department. Postreduction
radiographs are shown in Figures 14b and 14c. When the PIP joint is put through a
range of motion, the middle phalanx redislocates when fully extended. What is the
next most appropriate step in management? Review Topic
1 Extension block splinting for 3 weeks
2 Volar plate arthroplasty

3 Open reduction and internal fixation of the middle phalanx base

4 Superficialis tendon repair through bone tunnels

5 Dynamic external fixation

PREFERRED RESPONSE 1

The patient sustained a dorsal fracture-dislocation of the PIP joint. This is a very
common injury and treatment options are variable. When there is no fracture of the
base of the middle phalanx, extension block splinting is the best option. When a
fracture is present, the stability of the fracture and the proportion of the base of the
middle phalanx that is involved must be evaluated. If the fracture surface is less than
25% of the base and the joint reduces adequately in flexion, then extension block
splinting is the best treatment option. When more than 25% of the joint is involved
and/or the joint does not reduce completely in flexion, then surgical treatment is
indicated. Dynamic external fixation is an effective and less invasive option for
moderate injuries. Open repair or volar plate arthroplasty is typically reserved for
severe injuries of more than 50% of the joint surface. In this patient, less than 25% of
the joint surface is involved. The postreduction radiographs show an excellent
congruent reduction. Because the PIP joint redislocates only in full extension, a PIP
joint extension block splint should be applied (40-degree block) and the patient should
be allowed to flex the finger. Over the next 3 weeks, the extension block can be
reduced 10 degrees per week.
(SAE10BS.59) Conduction velocity changes in peripheral nerve chronic compression
syndromes result primarily from which of the following actions? Review Topic

1 Wallerian degeneration distal to the compression

2 Wallerian degeneration proximal to the compression

3 Transection of individual axons

4 Schwann cell proliferation and apoptosis

5 Apoptosis of the neuronal cell body

PREFERRED RESPONSE 4

Whereas early literature proposed Wallerian degeneration, recent studies have found
that chronic compression of the peripheral nerve induces Schwann cell turnover,
including both proliferation and apoptosis without axonal pathology. Additionally, the
Schwann cell phenotype becomes less promyelogenic and upregulates proregenerative
molecules such as vascular endothelial growth factor (VEGF). Wallerian degeneration
is a process that results from separating the axon from the cell body and the hallmark
is granular disintegration of the axonal cytoskeleton that is triggered by increased
axoplasmic calcium.

(OBQ07.92) A 56-year-old right hand dominant male presents to your office


complaining of right thumb pain worsened with pincer grip and using his mobile
phone. He is a writer, and is having difficulty holding his pen. Radiographs from this
visit are shown in Figure A. Compared with trapeziectomy alone, which of the
following treatment options is likely to result in superior pain relief and improvement
of key-pinch strength? Review Topic

1 Trapeziometacarpal corticosteroid injection followed by aggressive occupational therapy


2 Trapeziectomy with interpositional palmaris longus arthroplasty

3 Trapeziectomy, interpositional arthroplasty, and palmar oblique ligament reconstruction


using flexor carpi radialis autograft

4 Partial trapeziectomy with capsular interpositional arthroplasty

5 None of the above


PREFERRED RESPONSE 5

This patient has symptomatic basal joint arthritis with radiographic evidence of
pantrapezial arthritis. Simple trapeziectomy has been shown to provide pain relief and
improvement of key-pinch strength that is comparable to trapeziectomy plus
interpositional arthroplasty.

Definitive surgical management of basal joint arthritis commonly involves excision of


the diseased trapezium with concomitant interpositional arthroplasty at the
carpometacarpal joint in an effort to mantain the height of the metacarpal. This is
commonly done with flexor carpi radialis (FCR) or palmaris longus (PL) autograft.
Recent studies have called into question the need for interpositional arthroplasty,
suggesting that excision of the trapezium alone can provide non-inferior results.

Davis et. al. randomized 183 symptomatic trapeziometacarpal joints to one of three
procedures: trapeziectomy alone, trapeziectomy with palmaris longus interpositional
arthroplasty, or trapeziectomy with FCR interpositional arthroplasty and
reconstruction of the palmar oblique ligament. For all patients, the thumb metacarpal
was percutaneously pinned to the distal pole of the scaphoid to maintain the height of
the digit. Patients were evaluated at three and 12 months post-operatively. At both
time-points, they found no difference between groups with respect to subjective
accounts of pain, function, stiffness, and weakness. Objective measures of thumb key-
pinch strength were no different at either time point. The authors concluded that there
may be no benefit to ligament reconstruction or tendon interposition in the short term.

Li et. al. performed a systematic review of four randomized controlled trials and two
systematic reviews to evaluate outcomes of trapeziectomy with and without LRTI for
treatment of basal joint osteoarthritis. In their review, there were no statistically
significant differences in post-op grip strength, pinch strength, visual analog pain
scores, DASH scores, and complications. The authors concluded that both procedures
produced similar clinical results.

Raven et. al. performed a retrospective analysis of 54 patients who underwent one of
three procedures for basal joint osteoarthritis: resection arthroplasty, trapeziectomy
with tendon interposition, or trapeziometacarpal arthrodesis. The authors found
resection arthroplasty to be a simple procedure with long-term results pain and
functional outcomes comparable to trapeziectomy with tendon interposition.

Naram et. al. retrospectively reviewed 200 patients who underwent simple
trapeziectomy with or without LRTI and with or without Kirschner wire stabilization,
or a Weilby ligament reconstruction. They found that patients undergoing
trapeziectomy with LRTI or a Weilby procedure had a greater incidence of
complications compared to trapeziectomy alone, including infection and reoperation.

Figure A is a plain radiograph demonstrating pantrapezial arthritis with the thumb


trapeziometacarpal joint being most significantly affected.

Incorrect Answers:
Answer 1: The patient has symptomatic debilitating basal joint arthritis with
radiographic obliteration of the trapeziometacarpal joint, therefore cortisone injection
would be unlikely to provide durable response.
Answer 2 and 3: Trapeziectomy with interpositional arthroplasty using both PL or
FCR are acceptable treatment options for trapeziometacarpal arthritis, however
neither option is superior to trapeziectomy alone.
Answer 4: Partial trapeziectomy with capsular interpositional arthroplasty has been
proposed for treating basal joint arthritis, however the results of this technique have
not been well studied and the indications are not well defined. In this patient,
radiographs demonstrates moderate STT arthritis, suggesting that complete
trapeziectomy would be a better solution than partial trapeziectomy at the
trapeziometacarpal joint.

(SAE11PA.76) A 31-year-old patient has had a left medial elbow mass for 1 month.
The mass has been increasing in size and has now become very painful and
erythematous. MRI scans are shown in Figures 76a and 76b. Laboratory studies show
an erythrocyte sedimentation rate of 49 mm/h (normal 0 to 20 mm/h) and C-reactive
protein level of 23 mg/L (normal 0 to 0.3 mg/L). Histology showed lymphoid tissue
and multiple necrotizing granulomas. What organism is responsible for this clinical
picture? Review Topic

1 Borrelia burgdorferi
2 Trichophyton tonsurans

3 Bartonella henselae

4 Mycobacterium avium

5 Corynebacterium minutissimum

PREFERRED RESPONSE 3

Cat scratch disease (CSD) is an important diagnosis for the orthopaedic surgeon to
consider in the differential diagnosis of soft-tissue masses adjacent to epitrochlear or
cervical lymph nodes. It is a soft-tissue tumor simulator and a high index of suspicion
is necessary in all patients with upper extremity or head and neck adenopathy and a
history of cat exposure. Although generally not required for diagnosis, cross-sectional
imaging will reveal a mass with surrounding edema in an area of lymphatic drainage.
A peripheral blood sample can be tested for Bartonella henselae - the offending
organism with this diagnosis. Classically the histology of these lesions when biopsied
will show multiple necrotizing granulomas. Mycobacterium avium is the only other
organism that would demonstrate a granulomatous reaction and the location is classic
for CSD. Borrelia burgdorferi is associated with Lyme disease. Mycobacterium avium
may be a source of immunocompromised infections in HIV patients. Trichophyton
tonsurans and corynebacterium minutissimum are not associated with orthopaedic
diseases.

(OBQ14.24) The cross section of a peripheral nerve is shown in Figure A. What are
the parts labeled A through F respectively? Review Topic

1 axon, myelin, perineurium, epineurium, endoneurium, node of Ranvier


2 axon, myelin, epineurium, perineurium, endoneurium, node of Ranvier

3 axon, endoneurium, epineurium, perineurium, myelin, node of Ranvier

4 axon, epineurium, perineurium, endoneurium, myelin, node of Ranvier

5 axon, perineurium, epineurium, endoneurium, myelin, node of Ranvier

PREFERRED RESPONSE 2

The labelled parts are axon, myelin, epineurium, perineurium, endoneurium, node of
Ranvier respectively.

Peripheral nerves are highly organized structures. The axon is the main conducting
structure. The myelin sheath surrounds the axon and increases conduction velocity.
The endoneurium is fibrous tissue around the axon that aids formation of the Schwann
cell tube.Fascicles are a group of axons and surrounding endoneurium. The
perineurium is connective tissue sheath around each fascicle is is the primary source
of tensile strength and elasticity. The epineurium is a layer around a group of fascicles
and cushions against external pressure.

Figure A and Illustration A show the hierarchical structure of a peripheral nerve.

Incorrect Answers:
Answers 1, 3, 4, 5: The myelin sheath surrounds the axon.

(SAE08PA.94) A 58-year-old woman has had a slowly progressing mass over the
distal interphalangeal (DIP) joint of her dominant hand with a worsening deformity of
her nail. She has no significant medical history but underwent bilateral knee
arthroplasties 1 year ago. Radiographs reveal a small osteophyte at the DIP joint
dorsally. A clinical photograph and a biopsy specimen are shown in Figures 76a and
76b. What is the most likely diagnosis? Review Topic

1 Metastatic lung carcinoma


2 Mucous cyst

3 Synovial sarcoma

4 Inclusion cyst

5 Felon abscess

PREFERRED RESPONSE 2

A mucous cyst is thought to be a ganglion arising from the DIP joint in patients with
osteoarthritis. They are frequently associated with nail deformities. Treatment
involves removal of the cyst with debridement of DIP joint osteophytes.
(SAE08UE.122) A 22-year-old college quarterback is tackled and sustains a reducible
first carpometacarpal dislocation. What is the recommended treatment? Review Topic

1 Closed reduction and casting

2 Closed reduction and percutaneous pinning

3 First carpometacarpal arthrodesis

4 Dorsal capsulodesis

5 Ligament reconstruction using tendon autograft

PREFERRED RESPONSE 5

When comparing closed reduction and pinning to ligament reconstruction, the


reconstruction group had slightly better abduction and pinch strength. The volar
oblique ligament usually tears off the first metacarpal in a subperiosteal fashion. In
this young patient, motion-sparing procedures are preferred.

(SAE11UE.64) What is the effect of performing a flexor tenosynovectomy with an


open carpal tunnel release for idiopathic carpal tunnel syndrome? Review Topic

1 Increased risk of nerve injury


2 Improved postoperative finger flexion
3 No added long-term clinical benefit versus open carpal tunnel release alone
4 Increased postoperative pain
5 Decreased recurrence of carpal tunnel syndrome

PREFERRED RESPONSE 3

(OBQ08.34) All of the following are contents of the carpal tunnel EXCEPT: Review
Topic

1 Flexor pollicis longus (FPL)


2 Flexor digitorum sublimis (FDS)
3 Flexor digitorum profundus (FDP)
4 Flexor carpi radialis (FCR)
5 Median nerve

PREFERRED RESPONSE 4
(SAE07SM.38) A 28-year-old woman fell on her right wrist while rollerblading 2
days ago. She was seen in the emergency department at the time of injury and was
told she had a sprain. Examination now reveals dorsal tenderness in the proximal
wrist but no snuffbox or ulnar tenderness. Standard wrist radiographs are normal.
What is the next most appropriate step in management? Review Topic

1 Arthroscopy of the wrist

2 CT of the wrist

3 PA clenched fist radiograph

4 Electromyography and nerve conduction velocity studies

5 AP and lateral radiographs of the forearm

PREFERRED RESPONSE 3

When considering the diagnosis of scapholunate ligament injury, standard


radiographic views of the hand will not always reveal widening of the scapholunate
gap. Although MRI may reveal injury to the ligaments, the PA clenched fist view can
be obtained in the office during the initial patient visit. Arthroscopy is not a first-line
diagnostic tool.

(SBQ13PE.89) Constriction ring syndrome, also known as amniotic band syndrome,


is a congenital disorder associated with which paediatric foot condition?
Review Topic

1 Equinovalgus foot
2 Clubfoot (Congenital talipes equinovarus)
3 Tarsal coalition
4 Congenital vertical talus
5 Polydactyly

PREFERRED RESPONSE 2

(OBQ12.262) A 28-year-old NFL running back complains of continued hand pain three days
following an injury sustained while being tackled. He was splinted on the field. He has
tenderness over the long finger metacarpal head, with subluxation of the extensor tendon
into the intermetacarpal area during active metacarpophalangeal joint flexion. A
representative MRI is shown in Figure A. What is the next best step in management of this
patient? Review Topic
1 Observation alone
2 Continued splinting in flexion
3 Continued splinting in extension
4 Open repair of the disrupted junctura tendinae
5 Open repair of the disrupted sagittal band

PREFERRED RESPONSE 5

(OBQ10.6) The parents of a newly adopted 3-year-old boy bring the child to the office for
evaluation of his thumb. A clinical photograph is provided in figure A. Which of the following
is the most important factor in determining thumb reconstruction versus ablation and
pollicization? Review Topic

1 Stability of the carpometacarpal joint

2 Functional ROM of the wrist

3 Functional ROM of the index, middle, ring, and small fingers

4 Skin contracture of the first web space

5 Absence of a thumb interphalangeal joint

PREFERRED RESPONSE 1
(OBQ08.19) A 2-year-old boy has the upper limb deformity seen in Figures A and B. All of the
following are associated with this condition EXCEPT? Review Topic

1 Fanconi's Anemia
2 Holt-Oram syndrome
3 VATER syndrome
4 VACTERL syndrome
5 Osteogenesis Imperfecta

PREFERRED RESPONSE 5

(OBQ14.58) A 45-year-old patient undergoes multiple debridements for necrotizing


fasciitis of the forearm, leaving the defect shown in Figure A. The surgeon plans free
fasciocutaneous flap transfer using an antegrade flap based on septocutaneous
perforators running in the intermuscular septum, as seen in the cadaveric dissection
in Figure B. At 4 months, the donor site incision has the appearance shown in Figure
C. Which vessel do these perforators originate from? Review Topic

1 Brachial artery

2 Posterior radial collateral artery

3 Recurrent interosseous artery

4 Posterior ulnar recurrent artery

5 Middle collateral artery


PREFERRED RESPONSE 2

(OBQ14.96) What clinical finding is characteristic of involvement of the natatory


ligament in Dupuytren's disease? Review Topic

1 Palmar pits
2 Metacarpophalangeal joint contracture
3 Web space contracture
4 Distal interphalangeal joint contracture
5 Tender pads over the dorsal aspect of the proximal interphalangeal joints

PREFERRED RESPONSE 3

(OBQ11.125) Figure A depicts a child with syndactyly. Following surgical treatment,


the most common complication involves which of the following? Review Topic

1 Nail plate
2 Nail bed
3 Web commissure
4 Radial digital nerve
5 Radial digital artery

PREFERRED RESPONSE 3

(SAE11UE.45) A 25-year-old man was involved in an altercation. Examination


reveals loss of active extension of the middle finger metacarpophalangeal (MCP)
joint. A diagnosis of sagittal band rupture is made. Which of the following is
considered the key diagnostic finding? Review Topic

1 Extensor lag of 30 degrees


2 Extensor lag of 60 degrees
3 Positive Bunnell intrinsic tightness test
4 Ability to maintain active extension of the interphalangeal joints
5 Ability to maintain MCP extension after passive extension
PREFERRED RESPONSE 5

(SAE11AN.35) Normal thumb flexor tendon kinematics are restored by repairing


which of the following pulleys when the A-2 is intact? Review Topic

1 Av-2
2 Av-1
3 Oblique or A-1
4 A-3
5 Palmar aponeurotic

PREFERRED RESPONSE 3

(OBQ06.14) A 28-year-old factory worker has his ring finger caught in the machinery
at work. A photograph of the injury is shown in Figure A. Which of the following
procedures will best supply coverage of the wound? Review Topic

1 Amputation through the proximal interphalangeal joint


2 Shortening of the distal phalanx, nail bed removal, and V-Y flap
3 Cross-finger flap
4 Groin flap
5 Thenar flap

PREFERRED RESPONSE 2

(OBQ05.91) A 29-year-old intravenous drug user undergoes irrigation and


debridement of a ring finger abscess. After adequate eradication of the infection, he is
left with the skin defect shown in Figure A. What is the most appropriate treatment at
this time? Review Topic
1 Local woundcare and healing by secondary intention
2 V-Y advancement flap
3 Thenar flap
4 Moberg flap
5 Cross-finger flap

PREFERRED RESPONSE 5

(OBQ08.90) Madelung's deformity of the distal radius is caused by which of the


following? Review Topic

1 Premature fusion of the distal radial ulnar joint


2 Physeal growth mismatch between the distal radius and ulna
3 Nutritional deficiency affecting the physeal zone of provisional calcification
4 Impaired growth of the volar and ulnar aspect of the distal radial physis
5 Unrecognized trauma

PREFERRED RESPONSE 4

(OBQ06.1) A 32-year-old male sustains a type IIIb open proximal third tibia fracture.
Four days after intramedullary nailing of the tibia, the wound is clean and ready for
coverage with a medial gastrocnemius rotational flap. What is the dominant arterial
blood supply to this flap? Review Topic

1 Superior and inferior genicular arteries


2 Anterior tibial artery
3 Posterior tibal artery
4 Sural artery
5 Saphenous artery

PREFERRED RESPONSE 4
(SAE08OS.58) A 21-year-old man reports wrist pain after a fall. He also reports that
he sustained a wrist sprain the previous year. Radiographs reveal a waist scaphoid
fracture with a nonunion, humpback deformity, but no evidence of osteonecrosis.
What is the best choice for surgical treatment? Review Topic

1 Volar approach, standard bone graft, Kirschner wire fixation


2 Volar approach, standard bone graft, screw fixation
3 Volar approach, vascularized bone graft, Kirschner wire fixation
4 Dorsal approach, vascularized bone graft, Kirschner wire fixation
5 Dorsal approach, standard bone graft, screw fixation

PREFERRED RESPONSE 2

(OBQ06.52) A 22-year-old rugby player presents with a mass at the base of his ring
finger 5 months after sustaining an injury while making a tackle. Physical
examination demonstrates a lack of active distal interphalangeal joint flexion, but full
passive range of motion of all joints of the ring finger. Radiographs are normal. What
is the most appropriate treatment to regain normal finger function? Review Topic

1 excision of the palmar mass and 2-stage tendon grafting


2 excision of the palmar mass and single stage tendon grafting
3 excision of the palmar mass and distal interphalangeal joint fusion
4 active silicone rod implantation
5 flexor digitorum profundis repair

PREFERRED RESPONSE 1

(OBQ07.218) A 27-year-old man falls on his hand at work. He notices an immediate


deformity of his ring finger. Radiographs are provided in Figure A. Which of the
following is the most appropriate initial treatment? Review Topic

1 Closed reduction, buddy taping, and early motion to prevent stiffness


2 Closed reduction and full time extension splinting
3 Open reduction and repair of the central slip of the extensor tendon
4 Open reduction and repair of the volar plate
5 Amputation and immediate return to work

PREFERRED RESPONSE 2
(OBQ13.143) A 17-year-old boy presents with pain in his right elbow for 2 years and
limitation in elbow motion bilaterally. He denies any pain or discomfort in his left
elbow. He reports no history of trauma to either elbow. He has had two courses of
physical therapy, but has noted no noticeable improvement in pain or motion.
Examination demonstrates no elbow tenderness on palpation, and there are no
neurological deficits. Manual reduction is unsuccessful. The range of motion of both
elbows is shown in Figure A. Radiographs of left and right elbow are shown in Figure
B and C respectively. What is the most appropriate treatment plan for the right and
left elbow? Review Topic

1 Bilateral open reduction and application of a hinged external fixator to both elbows
2 Radial head resection of the right elbow and non-operative management of the left elbow.

3 Bilateral radial head arthroplasty

4 Physical therapy and splinting to both elbows

5 Radial head resection and interposition arthroplasty for the right elbow and radial head
resection alone for the left elbow

PREFERRED RESPONSE 2

This patient has bilateral congenital radial head dislocation (CRHD). The right side is
symptomatic with significant loss of motion. The left is asymptomatic with minimal
loss of active motion. Therefore the most appropriate treatment is radial head
resection of the right elbow and non-operative management of the left elbow.

It is important to differentiate CRHD from traumatic dislocation. Clinical features of


CRHD include bilateral involvement, presence at birth, other congenital anomalies,
familial occurrence, irreducible by closed methods, and lack of a history of trauma.
Radiological features include dome-shaped radial head and hypoplastic capitellum,
relatively short ulna or long radius, deficient trochlea, prominent medial epicondyle,
grooving of the distal radius, and anterior curvature of the posterior outline of the
ulna.

Bengard et al. reviewed 10 surgically treated and 6 nonsurgically treated CRHD


patients. They found no change in flexion-extension and carrying angle
postoperatively, but forearm rotation was improved. Surgically treated patients had
significant improvement in elbow pain. Ultimately, >25% of patients had wrist pain
postop and this must be weighed in the decision process of treatment. They
recommend radial head excision as an effective intervention in selected patients with
significant elbow pain.

Figure A is a table showing moderately diminished ROM of the right elbow, and
minimally reduced ROM of the left elbow. Figures B and C both show posterior
dislocation of the radial head (a line along the long axis of the radius should intersect
the capitellum in all views).

Incorrect Answers:
Answer 1: Open reduction is not reliable because of a high likelihood of recurrence of
dislocation (even if combined with ulnar/radial osteotomy or annular ligament
reconstruction) especially in the setting of a hypoplastic/absent capitellum, dome
shaped radial head, and a relatively longer radius.
Answer 3: Bilateral radial head arthroplasty is not indicated. Prosthetic radial head
replacement may lead to recurrent dislocation because of an under-developed
capitellum and angulation of the radial head away from the capitellum (non-collinear).
Answer 4: Physical therapy is unlikely to improve pain and ROM of the right elbow,
and unlikely to improve ROM in the left elbow.
Answer 5: Ulnohumeral distraction and interposition arthroplasty (DIA, distraction
interposition arthroplasty) may be an option for ulnohumeral arthritis. Interposition
arthroplasty is not indicated for radial head disease.

(OBQ14.246) A 12-year-old boy crushes his middle finger in a car door. He presents
with pain and swelling of the finger. Examination reveals an apparent mallet
deformity. The nail plate is not displaced superficial to the eponychial fold and there
is no evidence of a skin or nail bed laceration. Radiographs are shown in Figures A
and B. With the distal interphalangeal joint immobilized in extension, the distal
phalanx demonstrates a 40 degree arc of mobility at the fracture site under
fluoroscopy. What is the next best step? Review Topic
1 Mallet splint in situ
2 Buddy taping to the ring finger

3 Nail plate removal, nail bed repair, removal of interposed soft tissue, open reduction,
distal interphalangeal joint-sparing pin fixation

4 Nail plate elevation and replacement, open reduction, tendon repair, distal
interphalangeal joint-sparing mini screw fixation

5 Closed reduction and pin fixation across the distal interphalangeal joint

PREFERRED RESPONSE 5

This patient has a closed, unstable Seymour fracture. Closed injuries may be treated
with closed reduction alone. Unstable injuries should be treated with pinning across
the DIPJ.

Seymour fractures include Salter Harris I, II, and metaphyseal fractures 2mm distal to
the physis. When treating the fracture, the nail should be preserved if possible.
CLOSED fractures are treated with closed reduction (by hyperflexion) and splinting,
or pinning (if the fracture is unstable). If reduction is difficult, one can assume tissue
interposition and nail plate elevation and removal of soft tissue is indicated (similar to
treatment for OPEN fractures). OPEN fractures (with elevation of the nail plate
superficial to the eponychial fold) are treated with nail plate elevation, removal of
interposed soft tissue (usually germinal matrix), pinning, and nailbed repair. Closed
treatment of an open Seymour fracture puts patients at high risk of both osteomyelitis
and physeal arrest

Krusche-Mandl et al. retrospectively reviewed the treatment of 24 Seymour fractures.


Nine were treated with closed reduction and splinting, 9 with debridement, open
reduction, nail fixation and splinting, 5 with debridement, open reduction, nail
fixation and pinning, and 1 with debridement, open reduction and nail removal. They
found that 23/24 regained full motion but 11 had nail growth disturbances. They
recommend reduction and splinting for closed fractures, and reduction, debridement,
and K-wire fixation for instability in open injuries.

Abzug et al. reviewed Seymour fractures. They advocate a more aggressive approach
- surgical management for both open and closed injuries to ensure that there is no
interposed tissue in the fracture site to reduce the risk of physeal arrest, nail
deformity, and chronic osteomyelitis.

Figures A and B are lateral and AP radiographs showing a typical Seymour fracture
(Salter Harris II). Illustration A shows the Krusche-Mandl algorithm for treatment of
closed and open injuries.

Incorrect Answers:
Answer 1: Splinting alone without reduction will likely lead to physeal growth
deformity and nail growth disturbances.
Answer 2: Buddy taping provides inadequate splintage. Buddy taping is not used for
the distal phalanx.
Answer 3: DIPJ sparing fixation with a K wire is not feasible because of limited
purchase in the epiphyseal plate. While it would not be incorrect to consider nail plate
elevation and splinting, open reduction and inspection for interposed tissue in the
fracture is indicated if there was suspicion of interposed tissue. In this case, fixation of
K-wires should span the DIPJ.
Answer 4: There is no tendon disruption in this injury. The mallet posture occurs
because the terminal extensor slip and the volar plate only insert into the epiphysis
(extension force), whereas the FDP spans the epiphysis and inserts on the metaphysis
(flexion force). Screws are not used in this fixation because of the small size
(pediatric patient), fear of shutting down the physis, inadequate epiphyseal purchase,
and difficult removal in a child.

(SAE11UE.55) A 22-year-old motorcyclist sustains open fractures to the left radial


shaft and second and third metacarpals with exposed extensor tendon and bone. The
fractures are approached via the dorsal open wounds of the forearm and hand with no
additional incisions made. The radiograph and clinical photograph of the remaining
defect in the hand are shown in Figures 55a and 55b. The remaining wound can be
most appropriately covered with which of the following? Review Topic
1 Split-thickness skin grafting
2 Posterior interosseous rotational flap

3 Radial forearm rotational flap

4 Groin flap

5 Free lateral arm flap

PREFERRED RESPONSE 3

After adequate debridement, there is exposed bone, tendon, and hardware. Split-thickness
skin grafting over exposed tendon will not have a viable bed to support the graft. The
tendons would not have healthy surrounding tissue, resulting in poor tendon gliding. The
dorsal wound has disrupted the posterior interosseous artery that runs in the septum
between the extensor digiti minimi and the extensor carpi ulnaris. Following the
reconstructive ladder, the radial forearm rotational flap accomplishes wound coverage with
a local flap rather than a groin flap (a distant flap) or a lateral arm flap (microvascular free
tissue transfer

You might also like